You are on page 1of 114

Foot and Ankle Scored and Recorded Self-Assessment Examination 2015

Answer Book

0
Foot and Ankle Scored and Recorded Self-Assessment Examination 2015

1
Foot and Ankle Scored and Recorded Self-Assessment Examination 2015

Examination Center
Foot and Ankle Scored and Recorded Self-Assessment Examination 2015

Figure 1b

Figure 1a

CLINICAL SITUATION FOR QUESTIONS 1 THROUGH 4

Figures 1a and 1b are the radiographs of a 17-year-old boy who planted his right
foot while running and experienced immediate pain and an inability to bear
weight.

Question 1 of 100

This injury is best treated with

1- closed reduction and percutaneous pinning.


2- a short-leg cast with closed reduction.
3- open reduction and internal fixation (ORIF) with Kirschner wires.
4- ORIF with screw and/or plate fixation.

PREFERRED RESPONSE: 4- ORIF with screw and/or plate fixation.

2
Foot and Ankle Scored and Recorded Self-Assessment Examination 2015

Question 2 of 100

The Lisfranc ligament extends from the

1- medial cuneiform to the first metatarsal bone.


2- medial cuneiform to the second metatarsal bone.
3- first metatarsal base to the second metatarsal base.
4- first metatarsal base to the medial cuneiform.

PREFERRED RESPONSE: 2- medial cuneiform to the second metatarsal bone.

Question 3 of 100

In Lisfranc injuries, the "fleck sign," when present, represents

1- calcification of the interosseous ligament.


2- a stable Lisfranc injury.
3- a small avulsion fracture of the first metatarsal base.
4- a small avulsion fracture of the second metatarsal base.

PREFERRED RESPONSE: 4- a small avulsion fracture of the second


metatarsal base.

Question 4 of 100

In the evaluation of Lisfranc injuries, which radiographic studies should routinely


be obtained?

1- MRI
2- Bilateral weight-bearing anteroposterior and lateral views of the foot with
obliques
3- Stress radiographs under anesthesia
4- CT scan with 3-dimensional images

PREFERRED RESPONSE: 2- Bilateral weight-bearing anteroposterior and


lateral views of the foot with obliques

3
Foot and Ankle Scored and Recorded Self-Assessment Examination 2015

DISCUSSION

It is estimated that as many as 20% of Lisfranc injuries are missed on initial


radiographic examination. Weight-bearing bilateral radiographs should be
performed routinely. CT scan, MRI, and stress radiographs performed under
anesthesia may be needed in select cases. The Lisfranc ligament stabilizes the
midfoot and consists of the dorsal and plantar oblique ligaments and the strong
interosseous ligaments. All 3 extend from the base of the second metatarsal to the
medial cuneiform. The “fleck sign" is a small avulsion fracture at the medial base
of the second metatarsal, representing an avulsion of the Lisfranc ligament. The
current treatment recommendation for displaced Lisfranc subluxations and
dislocations is to perform ORIF with rigid fixation using either screws or plates
and screws. Kirschner wire fixation may lead to recurrence after pin removal.
Closed reduction and casting alone cannot permanently reduce the dislocation.

RECOMMENDED READINGS

Clanton TO, Waldrop III NE. Athletic injuries to the soft tissues of the foot and
ankle. In: Coughlin MJ, Saltzman CL, Anderson RB, eds. Mann's Surgery of the
Foot and Ankle. Vol 2. 9th ed. Philadelphia, PA: Elsevier-Saunders; 2014:1531-
1687.

Karges DB. Foot trauma. In: Cannada LK, ed. Orthopaedic Knowledge Update
11. Rosemont, IL: American Academy of Orthopaedic Surgeons; 2014:631-643.

4
Foot and Ankle Scored and Recorded Self-Assessment Examination 2015

Foot and Ankle Scored and Recorded Self-Assessment Examination 2015

Figure 5a Figure 5b

CLINICAL SITUATION FOR QUESTIONS 5 THROUGH 8

Figures 5a and 5b are the clinical photograph and AP radiograph of a 55-year-old


woman who has a painful right forefoot deformity. There is no history of
antecedent trauma. The 1-2 intermetatarsal angle is 17 degrees, and the hallux
valgus angle is 35 degrees.

Question 5 of 100

The second-toe deformity is most accurately described as

1- hammer toe.
2- claw toe.
3- curly toe.
4- crossover toe.

PREFERRED RESPONSE: 4- crossover toe.

5
Foot and Ankle Scored and Recorded Self-Assessment Examination 2015

Question 6 of 100

The patient's painful great-toe deformity is best treated with

1- proximal metatarsal bunionectomy.


2- simple bunionectomy.
3- cheilectomy.
4- resection of base of proximal phalanx.

PREFERRED RESPONSE: 1- proximal metatarsal bunionectomy.

Question 7 of 100

Disruption of which anatomic structure is necessary for the second-toe pathology


to occur?

1- Collateral ligaments
2- Intrinsic flexor tendons
3- Extrinsic extensor tendons
4- Plantar plate

PREFERRED RESPONSE: 4- Plantar plate

Question 8 of 100

On the morning of surgery the patient reports in the preop area that she has
experienced skin breakdown over the second toe for 10 days. The extensor tendon
is disrupted with an exposed proximal interphalangeal joint. She has been
applying antibiotic ointment to the wound and denies fever or chills. What is the
best plan of care?

1- Continue with plans for bunion and crossover toe correction.


2- Cancel the surgery and send the patient home with empiric oral medications.
3- Inform the patient that surgical joint debridement is necessary and the elective
reconstructive surgery will be put on hold.
4- Swab the ulcer and admit the patient for peripherally inserted central catheter
line placement and intravenous antibiotics.

PREFERRED RESPONSE: 3- Inform the patient that surgical joint


debridement is necessary and the elective reconstructive surgery will be put on
hold.

6
Foot and Ankle Scored and Recorded Self-Assessment Examination 2015

DISCUSSION

The clinical photograph shows a hallux valgus and a crossover toe deformity. The
plantar plate must be damaged for a crossover toe deformity to develop. A
moderately severe hallux valgus deformity without arthritic change is best treated
with a bunionectomy with a proximal metatarsal osteotomy. The surgeon must
assume that the open joint is at least colonized and at significant risk for
postsurgical infection; consequently, it is best to cancel elective surgery, and
surgical debridement of soft tissue and bone with deep cultures is recommended.

RECOMMENDED READINGS

Kaz AJ, Coughlin MJ. Crossover second toe: demographics, etiology, and
radiographic assessment. Foot Ankle Int. 2007 Dec;28(12):1223-37. doi:
10.3113/FAI.2007.1223. PubMed PMID: 18173985.
View Abstract at PubMed

Chalayon O, Chertman C, Guss AD, Saltzman CL, Nickisch F, Bachus KN. Role
of plantar plate and surgical reconstruction techniques on static stability of lesser
metatarsophalangeal joints: a biomechanical study. Foot Ankle Int. 2013
Oct;34(10):1436-42. doi: 10.1177/1071100713491728. Epub 2013 Jun 17.
PubMed PMID: 23774466.
View Abstract at PubMed

Harper MC, Keller TS. A radiographic evaluation of the tibiofibular syndesmosis.


Foot Ankle. 1989 Dec;10(3):156-60. PubMed PMID: 2613128.
View Abstract at PubMed

7
Foot and Ankle Scored and Recorded Self-Assessment Examination 2015

Foot and Ankle Scored and Recorded Self-Assessment Examination 2015

Figure 9

Question 9 of 100

Figure 9 is the radiograph of a 24-year-old amateur marathon runner who has


ankle pain. She previously sustained a metatarsal stress fracture. In addition to
asking about her training routine and the type of footwear she uses, the
orthopaedic surgeon should inquire about this patient's history of nutrition and

1- genetics.
2- rheumatology.
3- menstruation.
4- cardiovascular health.

PREFERRED RESPONSE: 3- menstruation.

8
Foot and Ankle Scored and Recorded Self-Assessment Examination 2015

DISCUSSION

Several studies have reported an increased incidence of stress fractures in female


athletes, including fractures of the foot and ankle in runners. The female athlete
triad describes a condition involving decreased bone density, anorexia, and
amenorrhea. In addition to asking about this woman's exercise routine, the
orthopaedic surgeon should obtain a comprehensive menstrual and dietary history
in the context of multiple stress fractures. A review of genetics, rheumatology,
and cardiovascular disorders is less likely to generate an etiology.

RECOMMENDED READINGS

Kasser JR, ed. Orthopaedic Knowledge Update 5: Home Study Syllabus.


Rosemont, IL: American Academy of Orthopaedic Surgeons; 1996:96-99.

Arendt EA. Osteoporosis in the athletic female: Amenorrhea and amenorrheic


osteoporosis. In: Pearl AJ, ed. AOSSM: The Athletic Female. Champaign, IL:
Human Kinetics; 1993:41-59.

Brukner PD, Khan KM. Clinical Sports Medicine. Sydney: McGraw-Hill;


1991:17.

9
Foot and Ankle Scored and Recorded Self-Assessment Examination 2015

Foot and Ankle Scored and Recorded Self-Assessment Examination 2015

RESPONSES FOR QUESTIONS 10 THROUGH 13

1- Ankle replacement
2- Ankle fusion
3- Tibiotalocalcaneal fusion
4- Total contact cast
5- Intra-articular steroid injection

Match the appropriate treatment listed above with the patient scenario described
below.

Question 10 of 100

A 28-year-old woman with bone-on-bone ankle arthritis, little deformity, and


recalcitrant pain

1- Ankle replacement
2- Ankle fusion
3- Tibiotalocalcaneal fusion
4- Total contact cast
5- Intra-articular steroid injection

PREFERRED RESPONSE: 2- Ankle fusion

Question 11 of 100

A 56-year-old woman with diabetes, neuropathy, and an unbraceable ankle and


hindfoot deformity

1- Ankle replacement
2- Ankle fusion
3- Tibiotalocalcaneal fusion
4- Total contact cast
5- Intra-articular steroid injection

PREFERRED RESPONSE: 3- Tibiotalocalcaneal fusion

10
Foot and Ankle Scored and Recorded Self-Assessment Examination 2015

Question 12 of 100

A 72-year-old man with a previous contralateral ankle fusion, rheumatoid


arthritis, and 5 degrees of valgus; he has pursued nonsurgical treatment for 30
years and now has unrelenting pain

1- Ankle replacement
2- Ankle fusion
3- Tibiotalocalcaneal fusion
4- Total contact cast
5- Intra-articular steroid injection

PREFERRED RESPONSE: 1- Ankle replacement

Question 13 of 100

A 72-year-old man with diabetic neuropathy and 5 degrees of valgus talar tilt; he
has pursued nonsurgical treatment for 30 years and now has unrelenting pain

1- Ankle replacement
2- Ankle fusion
3- Tibiotalocalcaneal fusion
4- Total contact cast
5- Intra-articular steroid injection

PREFERRED RESPONSE: 2- Ankle fusion

DISCUSSION

Arthritis of the ankle and hindfoot can pose challenges. Depending upon patient
age, comorbidities, and alignment, a variety of surgical interventions may be
offered. A total ankle replacement may be considered for patients older than 60
years of age who have minimal misalignment and low-demand lifestyles. In all
other cases, ankle fusion must be considered. The nonsurgical care of ankle
arthritis includes anti-inflammatory medication, intra-articular steroid injections,
bracing with customized products such as the Arizona brace, or a molded foot and
ankle orthosis.

Patients with diabetes and Charcot arthropathy may be treated nonsurgically with
total-contact casting during acute and active or "hot" phases and accommodative

11
Foot and Ankle Scored and Recorded Self-Assessment Examination 2015

shoes during consolidation and stable or "cool" phases. When the patient has
recurrent ulcers or major anatomy changes, surgical intervention must be
considered. Tibiotalocalcaneal fusion helps to realign the foot and ankle and
make it more braceable in the setting of ankle and hindfoot Charcot disease.

RECOMMENDED READINGS

Queen RM, Adams SB Jr, Viens NA, Friend JK, Easley ME, Deorio JK, Nunley
JA. Differences in outcomes following total ankle replacement in patients with
neutral alignment compared with tibiotalar joint malalignment. J Bone Joint Surg
Am. 2013 Nov 6;95(21):1927-34. doi: 10.2106/JBJS.L.00404. PubMed PMID:
24196462.
View Abstract at PubMed

Nunley JA, Caputo AM, Easley ME, Cook C. Intermediate to long-term


outcomes of the STAR Total Ankle Replacement: the patient perspective. J Bone
Joint Surg Am. 2012 Jan 4;94(1):43-8. doi: 10.2106/JBJS.J.01613. PubMed
PMID: 22218381.
View Abstract at PubMed

Saltzman CL, Mann RA, Ahrens JE, Amendola A, Anderson RB, Berlet GC,
Brodsky JW, Chou LB, Clanton TO, Deland JT, Deorio JK, Horton GA, Lee TH,
Mann JA, Nunley JA, Thordarson DB, Walling AK, Wapner KL, Coughlin MJ.
Prospective controlled trial of STAR total ankle replacement versus ankle fusion:
initial results. Foot Ankle Int. 2009 Jul;30(7):579-96. doi:
10.3113/FAI.2009.0579. PubMed PMID: 19589303.
View Abstract at PubMed

Faraj AA, Loveday DT. Functional outcome following an ankle or subtalar


arthrodesis in adults. Acta Orthop Belg. 2014 Jun;80(2):276-9. PubMed PMID:
25090803.
View Abstract at PubMed

Grear BJ, Rabinovich A, Brodsky JW. Charcot arthropathy of the foot and ankle
associated with rheumatoid arthritis. Foot Ankle Int. 2013 Nov;34(11):1541-7.
doi: 10.1177/1071100713500490. Epub 2013 Jul 30. PubMed PMID: 23900228.
View Abstract at PubMed

12
Foot and Ankle Scored and Recorded Self-Assessment Examination 2015

Foot and Ankle Scored and Recorded Self-Assessment Examination 2015

Figure 14

Question 14 of 100

Figure 14 is a sagittal-cut MR image from the hindfoot of a 54-year-old woman


who has had plantar heel pain for 3 months. There is no history of trauma. Her
pain is worse when she rises and at the end of the day. Upon examination she has
localizable tenderness over the plantar medial tubercle of the calcaneus. The
Achilles is intact and nontender, and subtalar joint motion is full and painless. A
Tinel test result is negative. What is the most likely diagnosis?

1- Calcaneal stress fracture


2- Hindfoot coalition
3- Insertional Achilles tendonitis
4- Plantar fasciitis

PREFERRED RESPONSE: 4- Plantar fasciitis

DISCUSSION

Plantar fasciitis is inflammation of the plantar fascia at its insertion onto the
medial calcaneus. The T2-weighted sagittal MR image reveals thickening of the
plantar fascia with no evidence of a calcaneal stress fracture, coalition, or
inflammation of the insertion of the Achilles tendon.

RECOMMENDED READINGS

Lareau CR, Sawyer GA, Wang JH, DiGiovanni CW. Plantar and medial heel
pain: diagnosis and management. J Am Acad Orthop Surg. 2014 Jun;22(6):372-

13
Foot and Ankle Scored and Recorded Self-Assessment Examination 2015

80. doi: 10.5435/JAAOS-22-06-372. PubMed PMID: 24860133.


View Abstract at PubMed

Covey CJ, Mulder MD. Plantar fasciitis: How best to treat? J Fam Pract. 2013
Sep;62(9):466-71. PubMed PMID: 24080555.
View Abstract at PubMed

14
Foot and Ankle Scored and Recorded Self-Assessment Examination 2015

Foot and Ankle Scored and Recorded Self-Assessment Examination 2015

Figure 15c
Figure 15a Figure 16a
Figure 15b

Figure 16b Figure 16c Figure 17

CLINICAL SITUATION FOR QUESTIONS 15 THROUGH 18

Figures 15a through 15c are the initial injury radiographs of a 32-year-old man
who sustained a closed injury to his right lower extremity after a fall from a curb.
Initial examination reveals a swollen painful ankle with pain both medially and
laterally at the level of the malleoli.

15
Foot and Ankle Scored and Recorded Self-Assessment Examination 2015

Question 15 of 100

Following surgical stabilization and fixation of the distal fibula, what is the most
appropriate next step?

1- Place a transsyndesmotic screw.


2- Perform an open repair of the deltoid ligament.
3- Perform a stress examination of the syndesmosis.
4- Close the wound and apply a short-leg splint.

PREFERRED RESPONSE: 3- Perform a stress examination of the


syndesmosis.

Question 16 of 100

Figures 16a through 16c are the postsurgical radiographs taken 3 months after
surgical stabilization of the fracture and syndesmosis. The patient has no pain and
symmetrical range of motion to the contralateral lower extremity. What is the
most appropriate next step?

1- Removal of the transyndesmotic screw


2- Retention of hardware with progression of activity
3- Revision open reduction and internal fixation (ORIF) of the syndesmosis
4- Syndesmotic fusion

PREFERRED RESPONSE: 2- Retention of hardware with progression of


activity

Question 17 of 100

One year after surgical fixation of the ankle (Figure 17) the patient has persistent
pain within the ankle and wants the hardware removed. He should be counseled
that after hardware removal he should expect

1- significant resolution of his pain and discomfort.


2- high risk of postoperative fibular fracture.
3- improvement in the range of motion.
4- no significant change in his symptoms.

PREFERRED RESPONSE: 4- no significant change in his symptoms.

16
Foot and Ankle Scored and Recorded Self-Assessment Examination 2015

Question 18 of 100

Which complication is most common after syndesmotic fixation?

1- Synostosis of the distal tibiofibular joint


2- Failure of fixation with subsequent loss of reduction
3- Malreduction of the distal tibiofibular joint
4- Posttraumatic ankle arthrosis

PREFERRED RESPONSE: 3- Malreduction of the distal tibiofibular joint

DISCUSSION

The injury radiographs reveal a supination external rotation IV ankle fracture with
evidence of medial clear space widening exceeding 4 mm and an increase relative
to the superior tibiotalar clear space. This indicates injury to the deltoid ligament
and necessitates surgical reduction and fixation to restore and maintain ankle
stability. Following stabilization of the fibula, an intraoperative stress
examination of the syndesmosis such as an external rotation stress test under
fluoroscopy or lateral pull on the fibula (the Cotton test) should be performed to
determine the integrity of the syndesmosis. Radiographic evidence of tibiofibular
clear space widening, medial clear space widening with external rotation, and
lateral displacement of the distal fibula when pulled is consistent with
syndesmotic injury. In contrast to the presurgical stress test, once the fibula has
been reduced and stabilized lateral talar translation can occur only if the
syndesmosis is injured in addition to the deltoid ligament. Failure of the
syndesmotic screw without evidence of malalignment of the mortise and a pain-
free ankle are not indications for further surgery because these patients have
satisfactory outcomes when compared to those who have intact or removed
screws. Hardware removal following fibula ORIF is indicated when patients have
pain directly related to hardware prominence. Resolution of joint pain or stiffness
is not a reliable outcome following hardware removal. Although fibular fracture
can occur, this is a rare complication. Malreduction of the syndesmosis is the
most common complication following ORIF of the syndesmosis and is improved
with direct visualization; however, malreduction still may occur with direct
visualization.

RECOMMENDED READINGS

Manjoo A, Sanders DW, Tieszer C, MacLeod MD. Functional and radiographic


results of patients with syndesmotic screw fixation: implications for screw
removal. J Orthop Trauma. 2010 Jan;24(1):2-6. doi:
17
Foot and Ankle Scored and Recorded Self-Assessment Examination 2015

10.1097/BOT.0b013e3181a9f7a5. PubMed PMID: 20035170.


View Abstract at PubMed

Jenkinson RJ, Sanders DW, Macleod MD, Domonkos A, Lydestadt J.


Intraoperative diagnosis of syndesmosis injuries in external rotation ankle
fractures. J Orthop Trauma. 2005 Oct;19(9):604-9. PubMed PMID: 16247304.
View Abstract at PubMed

Stark E, Tornetta P 3rd, Creevy WR. Syndesmotic instability in Weber B ankle


fractures: a clinical evaluation. J Orthop Trauma. 2007 Oct;21(9):643-6. PubMed
PMID: 17921840.
View Abstract at PubMed

Brown OL, Dirschl DR, Obremskey WT. Incidence of hardware-related pain and
its effect on functional outcomes after open reduction and internal fixation of
ankle fractures. J Orthop Trauma. 2001 May;15(4):271-4. PubMed PMID:
11371792.
View Abstract at PubMed

18
Foot and Ankle Scored and Recorded Self-Assessment Examination 2015

Foot and Ankle Scored and Recorded Self-Assessment Examination 2015

Figure 19

Question 19 of 100

Figure 19 is the clinical photograph of a 54-year-old man who underwent a total


ankle replacement (TAR). Three weeks after surgery he has increasing pain and a
deep wound as seen in the photograph. What is the best next step?

1- Remove the total ankle and place an antibiotic spacer


2- Debride and exchange polyethylene
3- Perform a below-the-knee amputation
4- Convert to a fusion with an intercalary allograft

PREFERRED RESPONSE: 2- Debride and exchange polyethylene

DISCUSSION

The patient is 3 weeks out from TAR. The wound is erythematous, and the tendon
is visible. At 3 weeks this is an acute wound breakdown. The preferred treatment
is a return to the operating room, an exchange of the polyethylene because the
wound appears deep enough to go down to the joint, and a flap for coverage.
Removal of the total ankle and placement of an antibiotic spacer should be
considered in the settings of subacute (6 weeks postop) or chronic infection
following TAR. A below-the-knee amputation may be considered with a failed
salvage or a chronically infected TAR. Conversion to a fusion may be considered
in situations in which the wound bed is not infected. In this case, there is concern
for ongoing active infection, and an intercalary allograft is not appropriate.

RECOMMENDED READINGS

Cho EH, Garcia R, Pien I, Thomas S, Levin LS, Hollenbeck ST. An algorithmic
approach for managing orthopaedic surgical wounds of the foot and ankle. Clin
Orthop Relat Res. 2014 Jun;472(6):1921-9. doi: 10.1007/s11999-014-3536-7.
19
Foot and Ankle Scored and Recorded Self-Assessment Examination 2015

Epub 2014 Feb 28. PubMed PMID: 24577615.


View Abstract at PubMed

Gadd RJ, Barwick TW, Paling E, Davies MB, Blundell CM. Assessment of a
three-grade classification of complications in total ankle replacement. Foot Ankle
Int. 2014 May;35(5):434-7. doi: 10.1177/1071100714524549. Epub 2014 Feb 14.
PubMed PMID: 24532698.
View Abstract at PubMed

20
Foot and Ankle Scored and Recorded Self-Assessment Examination 2015

Foot and Ankle Scored and Recorded Self-Assessment Examination 2015

Figure 20a Figure 20b

Question 20 of 100

Figures 20a and 20b are the radiographs of a 56-year-old woman who runs a
horse farm. She has a 2-year history of increasing ankle pain and swelling without
previous treatment. Which treatment is most appropriate at this time?

1 - Nonsteroidal anti-inflammatory drugs (NSAIDs) and bracing


2 - Ankle fusion
3 - Ankle arthroscopy and cheilectomy
4 - Ankle distraction arthroplasty

PREFERRED RESPONSE: 1 - Nonsteroidal anti-inflammatory drugs


(NSAIDs) and bracing

DISCUSSION

This patient has end-stage ankle arthritis. A short course of NSAIDs may provide
pain and inflammation relief. Bracing with either an ankle-foot orthosis or
Arizona brace can reduce pain by offloading the ankle joint. Ankle fusion is a
reliable procedure for treatment of end-stage ankle arthritis and is especially
recommended for active people after it is determined that nonsurgical measures
no longer provide adequate relief. Arthroscopic debridement and cheilectomy
may be indicated for bony impingement and mild arthritis with little articular
cartilage loss. The long-term results of ankle distraction arthroplasty are not yet
well defined but likewise would be reserved for scenarios in which nonsurgical

21
Foot and Ankle Scored and Recorded Self-Assessment Examination 2015

measures no longer provide adequate relief. The patient must be able to wear a
thin-wire external fixator for 3 months.

RECOMMENDED READINGS

Abidi NA, Neufeld SK, Brage ME, Reese KA, Sabharwal S, Paley, D. Ankle
arthritis. In: Pinzur MS, ed. Orthopaedic Knowledge Update: Foot and Ankle 4.
Rosemont, IL: American Academy of Orthopaedic Surgeons; 2008:159-193.

Saltzman CL: Ankle arthritis, in Coughlin MJ, Mann RA, Saltzman CL (eds):
Surgery of the Foot and Ankle. Philadelphia, PA, Mosby Elsevier, 2007, vol 1, pp
929-932.

22
Foot and Ankle Scored and Recorded Self-Assessment Examination 2015

23
Foot and Ankle Scored and Recorded Self-Assessment Examination 2015

Foot and Ankle Scored and Recorded Self-Assessment Examination 2015

Figure 21

Question 21 of 100

Figure 21 is the intraoperative fluoroscopic image of a 40-year-old man who felt a


pop during a twisting injury to his right ankle. He underwent open reduction and
internal fixation (ORIF) of a bimalleolar ankle fracture. During the surgery the
medial and lateral malleoli fractures were reduced and rigidly was internally
fixed. Following fracture fixation, which additional test is recommended to
ensure mortise stability?

1- Thompson
2- Cotton
3- Squeeze
4- Anterior drawer

PREFERRED RESPONSE: 2- Cotton

DISCUSSION

Following ORIF of a known osseous injury, stress testing of the syndesmosis is


recommended, especially for pronation-external rotation injuries. The Cotton test
applies a laterally directed force to the fibula to assess for widening of the distal
tibiofibular joint space. A positive Cotton test result indicates that syndesmotic
stabilization is indicated. The Thompson test is used to determine Achilles tendon

24
Foot and Ankle Scored and Recorded Self-Assessment Examination 2015

integrity. The squeeze test is a clinical, not intraoperative, assessment of


syndesmotic injury. The anterior drawer test assesses the integrity of the anterior
talofibular ligament.

RECOMMENDED READINGS

Zalavras C, Thordarson D. Ankle syndesmotic injury. J Am Acad Orthop Surg.


2007 Jun;15(6):330-9. Review. PubMed PMID: 17548882.
View Abstract at PubMed

Pakarinen H, Flinkkilä T, Ohtonen P, Hyvönen P, Lakovaara M, Leppilahti J,


Ristiniemi J. Intraoperative assessment of the stability of the distal tibiofibular
joint in supination-external rotation injuries of the ankle: sensitivity, specificity,
and reliability of two clinical tests. J Bone Joint Surg Am. 2011 Nov
16;93(22):2057-61. doi: 10.2106/JBJS.J.01287. PubMed PMID: 22262376.
View Abstract at PubMed

25
Foot and Ankle Scored and Recorded Self-Assessment Examination 2015

Foot and Ankle Scored and Recorded Self-Assessment Examination 2015

Figure 22

CLINICAL SITUATION FOR QUESTIONS 22 THROUGH 25

Figure 22 is a lateral radiograph of a 56-year-old woman who arrives at the


emergency department with a midfoot injury that occurred 4 weeks ago. Her
medical history is positive for diabetes mellitus, hypertension, and dialysis-
dependent chronic renal failure. She is a household ambulator. Her midfoot is red,
swollen, and deformed. The redness decreases with elevation. Her white blood
cell count is 5600/µL (reference range [rr], 4500-11000/µL) and her erythrocyte
sedimentation rate is 15 mm/h (rr, 0-20 mm/h).

Question 22 of 100

What is the most appropriate diagnosis?

1- Osteomyelitis
2- Osseous tumor
3- Stress fracture
4- Charcot foot

PREFERRED RESPONSE: 4- Charcot foot

Question 23 of 100

What is the most appropriate treatment?

1- Bone biopsy
2- Total-contact casting
3- Double upright ankle foot orthosis
4- Empiric antibiotics

PREFERRED RESPONSE: 2- Total-contact casting

26
Foot and Ankle Scored and Recorded Self-Assessment Examination 2015

Question 24 of 100

Glucose control assessment is best achieved by ordering which blood test?

1- Random glucose
2- Fasting glucose
3- Hemoglobin A1C
4- Prealbumin

PREFERRED RESPONSE: 3- Hemoglobin A1C

Question 25 of 100

The mechanism for the osseous destruction is attributable to

1- avascular necrosis.
2- tumor invasion.
3- hypervascularity.
4- infection.

PREFERRED RESPONSE: 3- hypervascularity.

DISCUSSION

This scenario is a classic example of the development of Charcot foot. A red,


swollen, deformed foot without ulceration suggests neuroarthropathy. Normal
inflammatory marker findings, no history of fever or chills, and radiographs
demonstrating bone loss support the diagnosis. Limb elevation with dramatic
reduction in erythema is also characteristic of this disease process and does not
occur with infection. Total-contact casting is the cornerstone of treatment for
acute Charcot disease. Hemoglobin A1C is an indicator of glucose averaged over
a 3-month period, providing the most reliable indication of a patient's ongoing
glucose control. The pathophysiology of bone destruction is believed to be
hypervascularity of bone. Infection and Charcot disease may develop
simultaneously, but the combination is rare.

RECOMMENDED READINGS

Kaynak G, Birsel O, Güven MF, Ogüt T. An overview of the Charcot foot


pathophysiology. Diabet Foot Ankle. 2013 Aug 2;4. doi:

27
Foot and Ankle Scored and Recorded Self-Assessment Examination 2015

10.3402/dfa.v4i0.21117.Print 2013. PubMed PMID: 23919113.


View Abstract at PubMed

Pinzur MS, Lio T, Posner M. Treatment of Eichenholtz stage I Charcot foot


arthropathy with a weightbearing total contact cast. Foot Ankle Int. 2006
May;27(5):324-9. PubMed PMID: 16701052.
View Abstract at PubMed

28
Foot and Ankle Scored and Recorded Self-Assessment Examination 2015

Foot and Ankle Scored and Recorded Self-Assessment Examination 2015

Question 26 of 100

Contracture of which structure causes hammertoe deformity?

1- Extensor digitorum longus tendon


2- Extensor digitorum brevis tendon
3- Flexor digitorum longus tendon
4- Flexor digitorum brevis tendon

PREFERRED RESPONSE: 3- Flexor digitorum longus tendon

DISCUSSION

A patient with a flexible hammertoe deformity has the deformity while standing,
but practically no deformity when seated with the foot in equinus. The
metatarsophalangeal joint is not involved. The deformity is created by contracture
of the flexor digitorum longus tendon.

RECOMMENDED READINGS

Coughlin MJ. Lesser toe abnormalities. Instr Course Lect. 2003;52:421-44.


Review. PubMed PMID: 12690869.
View Abstract at PubMed

Couglin MJ. Lesser toe deformities. In: Coughlin MJ, Mann RA, Saltzman CL,
eds. Surgery of the Foot and Ankle. Vol 1. 8th ed. Philadelphia, PA: Mosby
Elsevier; 2007:363-464.

29
Foot and Ankle Scored and Recorded Self-Assessment Examination 2015

Foot and Ankle Scored and Recorded Self-Assessment Examination 2015

Question 27 of 100

A 10-year-old boy reports heel pain with sporting activities. An examination


demonstrates gastrocnemius contracture and tenderness at the calcaneal
apophysis. Radiographs are unremarkable. What is the best next step?

1- MRI
2- Surgical intervention
3- Activity modification
4- Cast immobilization

PREFERRED RESPONSE: 3- Activity modification

DISCUSSION

Sever disease, or calcaneal apophysitis, is best treated with activity modification


that includes rest, restriction from sports and running, and Achilles tendon
stretching exercises. The diagnosis is clinical (rendering MRI study unnecessary)
and the course is usually self-limited, obviating the need for surgery.
Occasionally, children with severe symptoms may benefit from a short period of
cast or fracture brace immobilization.

RECOMMENDED READINGS

Sullivan RJ. Adolescent foot and ankle conditions. In: Pinzur MD, ED.
Orthopaedic Knowledge Update: Foot and Ankle 4. Rosemont, IL: American
Academy of Orthopaedic Surgeons; 2008:47-55.

Feldman DS. Osteochondrosis. In: Spivak JM, Di Cesare PE, Feldman Ds, et al,
eds. Orthopaedic: A Study Guide. New York, NY: McGraw-Hill; 1999:765-766.

30
Foot and Ankle Scored and Recorded Self-Assessment Examination 2015

Foot and Ankle Scored and Recorded Self-Assessment Examination 2015

Figure 28

Question 28 of 100

Figure 28 is the radiograph of a 25-year-old soccer player who twisted her left
ankle 1 week ago. She has pain and swelling over the anterolateral ankle and
there is ecchymosis over the lateral ankle. She has these muscle group findings:
anterior tibial tendon-right 5/5, left 5/5; posterior tibial tendon-right 5/5, left 5/5;
peroneals-right 5/5, left 4/5; Achilles-right 5/5, left 5/5. What is the best next
diagnostic or treatment step?

1- Stress radiographs
2- Surgical resection of the fragment with lateral ligament reconstruction
3- Physical therapy for peroneal strengthening and proprioceptive training
4- Ankle arthroscopy for debridement

PREFERRED RESPONSE: 3- Physical therapy for peroneal strengthening and


proprioceptive training

31
Foot and Ankle Scored and Recorded Self-Assessment Examination 2015

DISCUSSION

Thousands of ankle sprains occur in the United States every day. Most affected
patients do not have serious sequelae associated with their injury. In this case, a
young athlete sprained her ankle. Her only area of tenderness is isolated to the
anterior talofibular ligament. She also has associated weakness. The radiograph
shows an os subfibulare; this is an entity that she likely was born with. There is
no indication of bony pain, and it is too soon to test for instability; consequently,
no further imaging is required. Considering the nature of the sprain and her
weakness, physical therapy with proprioceptive training and peroneal
strengthening would be most beneficial.

RECOMMENDED READINGS

Lephart SM, Pincivero DM, Giraldo JL, Fu FH. The role of proprioception in the
management and rehabilitation of athletic injuries. Am J Sports Med. 1997 Jan-
Feb;25(1):130-7. PubMed PMID: 9006708.
View Abstract at PubMed

McGuine TA, Keene JS. The effect of a balance training program on the risk of
ankle sprains in high school athletes. Am J Sports Med. 2006 Jul;34(7):1103-11.
Epub 2006 Feb 13. PubMed PMID: 16476915.
View Abstract at PubMed

Chun TH, Park YS, Sung KS. The effect of ossicle resection in the lateral
ligament repair for treatment of chronic lateral ankle instability. Foot Ankle Int.
2013 Aug;34(8):1128-33. doi: 10.1177/1071100713481457. Epub 2013 Mar 7.
PubMed PMID: 23471672.
View Abstract at PubMed

32
Foot and Ankle Scored and Recorded Self-Assessment Examination 2015

Foot and Ankle Scored and Recorded Self-Assessment Examination 2015

Figure 29a Figure 29b Figure 33

CLINICAL SITUATION FOR QUESTIONS 29 THROUGH 33

Figures 29a and 29b are the weight-bearing radiographs of a 49-year-old woman
who has had several months of increasing pain and deformity in her left foot. She
points to her plantar medial arch as her area of greatest pain; however, she also
has pain just distal to the tip of the distal fibula. Her pain worsens with walking or
navigating stairs. Upon examination she has a flexible unilateral pes planus
deformity with increased heel valgus and forefoot abduction. She is unable to
perform a single heel raise.

Question 29 of 100

Which primary underlying pathologic finding is causing her symptoms?

1- Spring ligament tear


2- Gastrocnemius contracture
3- Tarsal coalition
4- Posterior tibial tendon dysfunction

PREFERRED RESPONSE: 4- Posterior tibial tendon dysfunction

33
Foot and Ankle Scored and Recorded Self-Assessment Examination 2015

Question 30 of 100

Injury to which ligament is commonly seen in this condition?

1- Cervical
2- Calcaneonavicular (spring)
3- Deltoid
4- Interosseous

PREFERRED RESPONSE: 2- Calcaneonavicular (spring)

Question 31 of 100

In addition to physical therapy, what is the best course of treatment at this time?

1- Steroid injection
2- Platelet-rich plasma injection
3- Ankle-foot orthosis
4- Foot orthosis with a lateral post

PREFERRED RESPONSE: 3- Ankle-foot orthosis

Question 32 of 100

Which surgical procedure should be considered after 6 months of unsuccessful


nonsurgical treatment?

1- Calcaneal osteotomy with bone graft and flexor digitorum longus tendon
transfer
2- Subtalar fusion
3- Triple arthrodesis
4- Ankle arthrodesis

PREFERRED RESPONSE: 1- Calcaneal osteotomy with bone graft and flexor


digitorum longus tendon transfer

34
Foot and Ankle Scored and Recorded Self-Assessment Examination 2015

Question 33 of 100

Figure 33 is the preoperative photograph of the patient's forefoot with the heel
taken out of valgus. Which procedure will best address this forefoot deformity
(which cannot be passively corrected by the examiner)?

1- First tarsometatarsal joint fusion


2- Gastrocnemius musculotendonous recession
3- Subtalar arthroereisis
4- Dorsal opening-wedge osteotomy of the medial cuneiform

PREFERRED RESPONSE: 4- Dorsal opening-wedge osteotomy of the medial


cuneiform

DISCUSSION

The most common cause of acquired adult flatfoot deformity (AAFD) is


dysfunction of the posterior tibial tendon. Tearing of the calcaneonavicular
(spring) ligament and gastrocnemius contracture results from longer-standing
attenuation of the posterior tibial tendon. Tarsal coalitions typically cause rigid
flatfoot deformity. The calcaneonavicular ligament comprises superomedial and
inferomedial bands. More than 70% of patients with AAFD have tearing of the
superomedial band. Tearing of the inferior band is seen less commonly. Deltoid
ligament insufficiency can be seen in long-standing valgus foot deformity. Initial
treatment should start with ankle-foot orthosis bracing and physical therapy.

The radiographs reveal loss of arch, significant uncoverage of the talar head by
the navicular, and lack of significant arthritis. Fusion procedures are not indicated
considering the patient's flexible deformity and the absence of hindfoot arthritis.
Realignment osteotomy must be combined with flexor digitorum longus tendon
transfer to successfully alleviate this patient's symptoms. Lateral column
lengthening will correct the forefoot abduction and talonavicular subluxation. A
medial sliding osteotomy can achieve additional correction and decompress
subfibular impingement. A dorsal opening plantar flexion (Cotton) osteotomy of
the medial cuneiform is an adjunct procedure that is needed to balance the foot in
cases of residual forefoot varus, as seen in the clinical photograph.

RECOMMENDED READINGS

Pinney SJ, Lin SS. Current concept review: acquired adult flatfoot deformity.
Foot Ankle Int. 2006 Jan;27(1):66-75. Review. PubMed PMID: 16442033.
View Abstract at PubMed
35
Foot and Ankle Scored and Recorded Self-Assessment Examination 2015

Bluman EM, Title CI, Myerson MS. Posterior tibial tendon rupture: a refined
classification system. Foot Ankle Clin. 2007 Jun;12(2):233-49, v. Review.
PubMed PMID: 17561198.
View Abstract at PubMed

Haddad SL, Mann RA. Flatfoot deformity in adults. In: Coughlin MJ, Mann RA,
Saltzman CL, eds. Surgery of the Foot and Ankle. 8th ed. Philadelphia, PA:
Mosby Elsevier; 2007:1007-1085.

36
Foot and Ankle Scored and Recorded Self-Assessment Examination 2015

Foot and Ankle Scored and Recorded Self-Assessment Examination 2015

Question 34 of 100

Which stress fracture location is reported most frequently among ballet dancers?

1- Base of the second metatarsal


2- Base of the fifth metatarsal
3- Tibial sesamoid
4- Central third of the tarsal navicular

PREFERRED RESPONSE: 1- Base of the second metatarsal

DISCUSSION

Stress fractures are a frequent overuse injury among professional ballet dancers.
The most common location is at the proximal metaphyseal-diaphyseal junction of
the second metatarsal. Repetitive stress injuries and fractures of the tibial
sesamoid, tarsal navicular, and base of the fifth metatarsal occur among other
athletes.

RECOMMENDED READINGS

O'Malley MJ, Hamilton WG, Munyak J, DeFranco MJ. Stress fractures at the
base of the second metatarsal in ballet dancers. Foot Ankle Int. 1996
Feb;17(2):89-94. PubMed PMID: 8919407.
View Abstract at PubMed

Micheli LJ, Sohn RS, Solomon R. Stress fractures of the second metatarsal
involving Lisfranc's joint in ballet dancers. A new overuse injury of the foot. J
Bone Joint Surg Am. 1985 Dec;67(9):1372-5. PubMed PMID: 4077907.
View Abstract at PubMed

Gehrmann RM, Renard RL. Current concepts review: Stress fractures of the foot.
Foot Ankle Int. 2006 Sep;27(9):750-7. Review. PubMed PMID: 17038292.
View Abstract at PubMed

37
Foot and Ankle Scored and Recorded Self-Assessment Examination 2015

Foot and Ankle Scored and Recorded Self-Assessment Examination 2015

Figure 35

Question 35 of 100

Figure 35 is the radiograph of a 37-year-old woman who began having right


forefoot pain about 4 weeks ago after increasing her daily running mileage. She
denies any specific injury. Upon examination she has tenderness over the medial
forefoot with mild swelling. In addition to her activity level, what is the primary
etiology of the radiograph finding?

1- Osteoporosis
2- Hallux valgus deformity
3- Hallux rigidus
4- A relatively long second metatarsal

PREFERRED RESPONSE: 4- A relatively long second metatarsal

DISCUSSION

Stress fractures are the result of physiological bone response to increased stress.
Increased stress on bone triggers an increase in remodeling, which begins with
resorption of bone at the site of stress. Ongoing stress can overwhelm bone
38
Foot and Ankle Scored and Recorded Self-Assessment Examination 2015

strength, resulting in a fracture. In the foot this most commonly is seen in the
second metatarsal at the junction of the middle and distal thirds. Contributing
factors to increased loading of the second metatarsal include hallux valgus
(decreased hallux loading transfers to the second metatarsal head), hallux rigidus
(offloading of the hallux attributable to pain increases second metatarsal loading),
and a long second metatarsal (increased duration of contact during push-off in the
stance phase).

RECOMMENDED READINGS

Shindle MK, Endo Y, Warren RF, Lane JM, Helfet DL, Schwartz EN, Ellis SJ.
Stress fractures about the tibia, foot, and ankle. J Am Acad Orthop Surg. 2012
Mar;20(3):167-76. doi: 10.5435/JAAOS-20-03-167. Review. PubMed PMID:
22382289.
View Abstract at PubMed

Donahue SW, Sharkey NA. Strains in the metatarsals during the stance phase of
gait: implications for stress fractures. J Bone Joint Surg Am. 1999
Sep;81(9):1236-44. PubMed PMID: 10505520.
View Abstract at PubMed

39
Foot and Ankle Scored and Recorded Self-Assessment Examination 2015

Foot and Ankle Scored and Recorded Self-Assessment Examination 2015

Question 36 of 100

Which radiographic abnormality most accurately serves as a predictor of ankle


syndesmosis disruption?

1- Medial clear space equal to the superior clear space on the anteroposterior (AP)
view
2- Tibiofibular overlap exceeding 6 mm on the AP view
3- Tibiofibular clear space exceeding 6 mm on AP view
4- Talocrural angle symmetric to the opposite side

PREFERRED RESPONSE: 3- Tibiofibular clear space exceeding 6 mm on AP


view

DISCUSSION

Normal syndesmotic relationships include a tibiofibular clear space smaller than 6


mm on both AP and mortise views. In a 1989 cadaveric study by Harper and
Keller, a tibiofibular clear space exceeding 6 mm on both the AP and mortise
views was the most reliable predictor of early syndesmotic widening. Tibiofibular
overlap is measured 1 cm proximal to the plafond. Normal values exceed 6 mm or
42% of the width of the fibula on the AP view, or 1 mm on the mortise view.
Proximal fibula fracture can occur in isolation without syndesmotic injury,
frequently after direct trauma. The medial clear space is the distance between the
lateral border of the medial malleolus and the medial border of the talus and is
measured at the level of the talar dome. In the mortise view with the ankle in
neutral dorsiflexion, the medial clear space should be equal to or smaller than the
superior clear space between the talar dome and the tibial plafond. ?A normal
medial clear space may be present with syndesmotic injury and consequently
lacks sensitivity and specificity.

RECOMMENDED READINGS

Zalavras C, Thordarson D. Ankle syndesmotic injury. J Am Acad Orthop Surg.


2007 Jun;15(6):330-9. Review. PubMed PMID: 17548882.
View Abstract at PubMed

Wuest TK. Injuries to the Distal Lower Extremity Syndesmosis. J Am Acad


Orthop Surg. 1997 May;5(3):172-181. PubMed PMID: 10797219.
View Abstract at PubMed

40
Foot and Ankle Scored and Recorded Self-Assessment Examination 2015

Harper MC, Keller TS. A radiographic evaluation of the tibiofibular syndesmosis.


Foot Ankle. 1989 Dec;10(3):156-60. PubMed PMID: 2613128.
View Abstract at PubMed

41
Foot and Ankle Scored and Recorded Self-Assessment Examination 2015

Foot and Ankle Scored and Recorded Self-Assessment Examination 2015

Figure 38a Figure 38b


Figure 39a Figure 39b

Figure 40a Figure 40b


Figure 39c Figure 39d

CLINICAL SITUATION FOR QUESTIONS 37 THROUGH 40

A 41-year-old man sustained a twisting injury while running up stairs 4 weeks


ago. He was treated in an ankle brace and has been bearing weight since the
injury occurred. He has no history of ankle problems, but he now has ankle pain,
swelling, and instability. The pain is aggravated by stairs, and the instability is
worse on unlevel ground. Radiographs do not show a fracture.

42
Foot and Ankle Scored and Recorded Self-Assessment Examination 2015

Question 37 of 100

What is the appropriate treatment at this time?

1- Begin a structured proprioceptive-based rehabilitation program and use a brace


as needed.
2- Discontinue the brace and start aggressive range-of-motion physical therapy.
3- Use a short-leg weight-bearing cast for 4 weeks.
4- Maintain nonweight-bearing status on crutches for 4 weeks.

PREFERRED RESPONSE: 1- Begin a structured proprioceptive-based


rehabilitation program and use a brace as needed.

Question 38 of 100

Three months later this patient has continued swelling and giving-way episodes.
Figures 38a and 38b are his stress radiographs. This study indicates laxity in
which ligament?

1- Anterior talofibular
2- Calcaneal fibular
3- Posterior talofibular
4- Lateral talocalcaneal

PREFERRED RESPONSE: 1- Anterior talofibular

Question 39 of 100

The continued pain and instability 4 months after injury are likely related to
which finding on the presurgical MR images in Figures 39a through 39d?

1- Peroneal brevis tear


2- Osteochondral lesion of the talus
3- Loose body in the posterior ankle
4- Tear of the calcaneal-fibular ligament

PREFERRED RESPONSE: 2- Osteochondral lesion of the talus

43
Foot and Ankle Scored and Recorded Self-Assessment Examination 2015

Question 40 of 100

Figures 40a and 40b are this patient's intraoperative arthroscopic images. The
abnormality seen here illustrates which of the patient's clinical findings?

1- Instability on unlevel ground


2- Ankle pain and swelling
3- Anterior laxity on lateral radiographs
4- Osteochondral lesion of the talus on MR imaging

PREFERRED RESPONSE: 2- Ankle pain and swelling

DISCUSSION

Ankle sprains are the most common musculoskeletal injury; however, most of
these sprains do not progress to chronic instability. Initial injuries are treated with
RICE (rest, ice, compression, elevation), range of motion, weight bearing as
tolerated, and proprioceptive therapy. Lace-up ankle braces are most effective
during the subacute period after a sprain. Structured physical therapy focused on
proprioception is recommended for 6 weeks. Examination findings for ankle
ligament instability are unreliable because of associated subtalar joint motion.
Casting is not as effective as functional rehabilitation. Stress radiographs are
recommended, but a clear pathologic range of measurements is not defined.
Generalized ligament laxity can result in false-positive findings of instability;
therefore, contralateral stress radiographs are often necessary for comparison. The
difference in anterior drawer measurement between both ankles should not
exceed 5mm. Likewise, the difference in talar tilt measurement between both
ankles should be 5 or fewer degrees. Patients with mechanical symptoms, a joint
effusion, or continued pain may have an intra-articular pathology such as a loose
body or osteochondral lesion. Ankle instability can exist without ligamentous
laxity. Symptoms of chronic instability can result from osteochondral lesions of
talus, peroneal tendon pathology, loose bodies, anterior ankle impingement, and
fracture nonunions. Although there is not sufficient evidence to recommend
arthroscopy prior to all ligament reconstructions, arthroscopy is recommended
when other pathology is suspected.

RECOMMENDED READINGS

Colville MR. Surgical treatment of the unstable ankle. J Am Acad Orthop Surg.
1998 Nov-Dec;6(6):368-77. Review. PubMed PMID: 9826420.
View Abstract at PubMed

44
Foot and Ankle Scored and Recorded Self-Assessment Examination 2015

DiGiovanni CW, Brodsky A. Current concepts: lateral ankle instability. Foot


Ankle Int. 2006 Oct;27(10):854-66. Review. PubMed PMID: 17054892.
View Abstract at PubMed

Maffulli N, Ferran NA. Management of acute and chronic ankle instability. J Am


Acad Orthop Surg. 2008 Oct;16(10):608-15. Review. PubMed PMID: 18832604.
View Abstract at PubMed

45
Foot and Ankle Scored and Recorded Self-Assessment Examination 2015

Foot and Ankle Scored and Recorded Self-Assessment Examination 2015

Question 41 of 100

Which nerve is not included in a standard popliteal nerve block?

1- Sural nerve
2- Saphenous nerve
3- Superficial peroneal nerve
4- Deep peroneal nerve

PREFERRED RESPONSE: 2- Saphenous nerve

DISCUSSION

A standard popliteal nerve block is performed with the patient prone. The
injection aims for the area at, or close to, the peroneal and tibial nerves. The sural
nerve branches distal to the injection site, so this nerve and the superficial
peroneal, deep peroneal, and tibial nerves are covered with the injection. The
saphenous nerve is in an anteromedial location at knee level and is not close
enough to the area covered by the posterior injection to be included in the
analgesic effect.

RECOMMENDED READINGS

Varitimidis SE, Venouziou AI, Dailiana ZH, Christou D, Dimitroulias A, Malizos


KN. Triple nerve block at the knee for foot and ankle surgery performed by the
surgeon: difficulties and efficiency. Foot Ankle Int. 2009 Sep;30(9):854-9.
PubMed PMID: 19755069.
View Abstract at PubMed

Hromádka R, Barták V, Popelka S, Jahoda D, Pokorný D, Sosna A. [Regional


anaesthesia of the foot achieved from two cutaneous points of injection: an
anatomical study]. Acta Chir Orthop Traumatol Cech. 2009 Apr;76(2):104-9.
Czech. PubMed PMID: 19439129.
View Abstract at PubMed

Tran D, Clemente A, Finlayson RJ. A review of approaches and techniques for


lower extremity nerve blocks. Can J Anaesth. 2007 Nov;54(11):922-34. Review.
PubMed PMID: 17975239.
View Abstract at PubMed

46
Foot and Ankle Scored and Recorded Self-Assessment Examination 2015

Foot and Ankle Scored and Recorded Self-Assessment Examination 2015

Figure 42a Figure 42b

Figure 43a Figure 43b

CLINICAL SITUATION FOR QUESTIONS 42 THROUGH 44

Figures 42a and 42b are the radiographs of a 32-year-old man with an accessory
navicular, pes planovalgus deformity, and an associated gastrocnemius

47
Foot and Ankle Scored and Recorded Self-Assessment Examination 2015

contracture. He has been treated with custom orthotics and a physical therapy
program for several years and has progressed to stage II posterior tibial tendon
dysfunction (PTTD). This patient is now interested in surgery. Tendon
reconstruction with bony procedure to correct alignment, medializing calcaneal
osteotomy with lateral column lengthening, and a subtalar arthroereisis implant
are discussed with the patient.

Question 42 of 100

Which complication associated with subtalar arthroereisis devices for treatment of


flexible flatfoot deformity is most common?

1- Fracture of the talus


2- Osteonecrosis of the calcaneus
3- Persistent pain in the sinus tarsi
4- Attritional rupture of the flexor digitorum longus (FDL) tendon

PREFERRED RESPONSE: 3- Persistent pain in the sinus tarsi

Question 43 of 100

Figures 43a and 43b are the postsurgical radiographs. Which tendon transfer is
most appropriate for this patient's treatment?

1- Peroneus longus
2- Plantaris
3- Flexor hallucis longus
4- FDL

PREFERRED RESPONSE: 4- FDL

Question 44 of 100

Os naviculare is present in which percentage of normal feet?

1- 1% to 2%
2- 10% to 14%
3- 25% to 30%
4- 40% to 50%

PREFERRED RESPONSE: 2- 10% to 14%

48
Foot and Ankle Scored and Recorded Self-Assessment Examination 2015

DISCUSSION

Accessory navicular is found in 10% to 14% of normal feet, is generally


asymptomatic, and involves 3 radiographic types. Type I represents a small
ossicle embedded within the posterior tibial tendon, type II is larger with a
synchondrosis, and type III is fused to the navicular tuberosity. Approximately
50% of patients with symptoms have flexible flatfoot; however, os naviculare is
not directly associated with pes planovalgus deformity.

Subtalar arthroereisis describes the use of a sinus tarsi plug or implant to restrict
eversion of the subtalar joint. This surgical procedure has been used in
combination with tendon reconstruction for treatment of flexible flatfoot
deformity. Known complications of subtalar arthroereisis include persistent sinus
tarsi pain, foreign body reaction, implant failure, and osteonecrosis of the talus.

The FDL tendon travels within the same compartment adjacent to the posterior
tibial tendon and is the most commonly used tendon transfer for treatment of
stage II PTTD (strength characteristics are similar). The plantaris has inferior
tendon strength to the FDL, and the peroneus longus travels in a different
compartment than the FDL.

RECOMMENDED READINGS

Sullivan RJ. Adolescent foot and ankle conditions. In: Pinzur MD, ED.
Orthopaedic Knowledge Update: Foot and Ankle 4. Rosemont, IL: American
Academy of Orthopaedic Surgeons; 2008:47-55.

Alvarez RG, Price J, Marini A, Turner NS, Kitaoka HB. Adult acquired flatfoot
deformity and posterior tibial tendon dysfunction. In: Pinzur MD, ED.
Orthopaedic Knowledge Update: Foot and Ankle 4. Rosemont, IL: American
Academy of Orthopaedic Surgeons; 2008:215-229.

Pinney SJ, Lin SS. Current concept review: acquired adult flatfoot deformity.
Foot Ankle Int. 2006 Jan;27(1):66-75. Review. PubMed PMID: 16442033.
View Abstract at PubMed

Viladot R, Pons M, Alvarez F, Omaña J. Subtalar arthroereisis for posterior tibial


tendon dysfunction: a preliminary report. Foot Ankle Int. 2003 Aug;24(8):600-6.
PubMed PMID: 12956565.
View Abstract at PubMed

49
Foot and Ankle Scored and Recorded Self-Assessment Examination 2015

Foot and Ankle Scored and Recorded Self-Assessment Examination 2015

Figure 45a Figure 45b Figure 45c

Figure 45d Figure 45e

Question 45 of 100

Figures 45a through 45c are the MR images of a 22-year-old woman who has had
6 months of ankle pain related to activities of daily living. She recently completed

50
Foot and Ankle Scored and Recorded Self-Assessment Examination 2015

a course of cast immobilization and protected weight bearing without symptom


resolution. Figures 45d and 45e are the intraoperative arthroscopy images after
minimal probing. What is the most appropriate treatment?

1- Ankle fusion
2- Arthroscopic debridement and drilling
3- Retrograde drilling and bone grafting
4- Malleolar osteotomy and osteochondral grafting

PREFERRED RESPONSE: 4- Malleolar osteotomy and osteochondral grafting

DISCUSSION

The MR images reveal a large cystic medial talar dome osteochondral lesion
(OCL) in a patient who has failed nonsurgical treatment. Ankle fusion is
inappropriate because the patient has an otherwise normal ankle. Arthroscopic
debridement and drilling are appropriate for smaller (< 1.5 cm sq) noncystic
lesions. Retrograde drilling and bone grafting is an option in the treatment of
cystic OCL if the cartilage surface is intact; however, intraoperative arthroscopy
images show that this patient's cartilage surface is unstable. Osteochondral
allografts and autografts are effective in the treatment of large cystic talar dome
OCLs but are not appropriate for the initial surgical treatment of smaller lesions
like this one.

RECOMMENDED READINGS

Hannon CP, Smyth NA, Murawski CD, Savage-Elliott I, Deyer TW, Calder JD,
Kennedy JG. Osteochondral lesions of the talus: aspects of current management.
Bone Joint J. 2014 Feb;96-B(2):164-71. doi: 10.1302/0301-620X.96B2.31637.
Review. PubMed PMID: 24493179.
View Abstract at PubMed

Easley ME, Latt LD, Santangelo JR, Merian-Genast M, Nunley JA 2nd.


Osteochondral lesions of the talus. J Am Acad Orthop Surg. 2010
Oct;18(10):616-30. Review. PubMed PMID: 20889951.
View Abstract at PubMed

51
Foot and Ankle Scored and Recorded Self-Assessment Examination 2015

Foot and Ankle Scored and Recorded Self-Assessment Examination 2015

Question 46 of 100

A 32-year-old woman has had progressive left foot pain over the first
metatarsophalangeal (MTP) joint. Footwear is becoming problematic. There is
full range of motion of the first MTP with medial eminence pain. Her weight-
bearing radiograph reveals a hallux valgus angle (HVA) of 35 degrees and a 1-2
intermetatarsal angle (IMA) of 10 degrees. What is the best next step?

1- Distal metatarsal osteotomy


2- Distal soft-tissue reconstruction
3- Proximal metatarsal osteotomy
4- Lapidus bunionectomy

PREFERRED RESPONSE: 1- Distal metatarsal osteotomy

DISCUSSION

Patients with painful progressive hallux valgus are surgical candidates.


Presurgical evaluation includes radiographic examination. The IMA between the
first and second metatarsals as well as the HVA must be measured. If the IMA is
smaller than 15 degrees and the HVA is smaller than 35 degrees, a distal
osteotomy is preferred. Distal soft-tissue reconstruction is only useful for IMAs
smaller than 11 degrees and HVAs smaller than 25 degrees. Proximal osteotomies
and the Lapidus bunionectomy are reserved for larger hallux valgus deformities
with IMAs exceeding 15 degrees and HVAs exceeding 35 degrees.

RECOMMENDED READINGS

Pentikainen I, Ojala R, Ohtonen P, Piippo J, Leppilahti J. Distal Chevron


Osteotomy: Preoperative Radiological Factors Contributing to Long-Term
Radiological Recurrence of Hallux Valgus. Foot Ankle Int. 2014 Sep 5. pii:
1071100714548703. [Epub ahead of print] PubMed PMID: 25192724.
View Abstract at PubMed

Fakoor M, Sarafan N, Mohammadhoseini P, Khorami M, Arti H, Mosavi S,


Aghaeeaghdam A. Comparison of Clinical Outcomes of Scarf and Chevron
Osteotomies and the McBride Procedure in the Treatment of Hallux Valgus
Deformity. Arch Bone Jt Surg. 2014 Mar;2(1):31-6. Epub 2014 Mar 15. PubMed
PMID: 25207310.
View Abstract at PubMed

52
Foot and Ankle Scored and Recorded Self-Assessment Examination 2015

Park YB, Lee KB, Kim SK, Seon JK, Lee JY. Comparison of distal soft-tissue
procedures combined with a distal chevron osteotomy for moderate to severe
hallux valgus: first web-space versus transarticular approach. J Bone Joint Surg
Am. 2013 Nov 6;95(21):e158. doi: 10.2106/JBJS.L.01017. PubMed PMID:
24196470.
View Abstract at PubMed

53
Foot and Ankle Scored and Recorded Self-Assessment Examination 2015

Foot and Ankle Scored and Recorded Self-Assessment Examination 2015

RESPONSES FOR QUESTION 47 THROUGH 50

1- Observation
2- Arizona brace
3- Medial arch support
4- Casting
5- Hindfoot fusion

Select the most appropriate initial treatment from the list above to address each of
the conditions described below.

Question 47 of 100

An 8-year-old boy with pes planus that reconstitutes with heel-rise; his mother
brought him in for evaluation because he seems to be "tripping a lot".

1- Observation
2- Arizona brace
3- Medial arch support
4- Casting
5- Hindfoot fusion

PREFERRED RESPONSE: 1- Observation

Question 48 of 100

A 37-year-old woman has had persistent right lateral ankle pain after sustaining a
minor sprain 5 months ago. She has a sense of instability on uneven ground.
Physical therapy has not helped. She is tender along the peroneal tendons and in
the sinus tarsi. She has a negative anterior drawer test result for the ankle and no
tenderness over the anterior lateral malleolus. She also has bilateral pes planus
that persists with heel rise.

1- Observation
2- Arizona brace
3- Medial arch support
4- Casting
5- Hindfoot fusion

PREFERRED RESPONSE: 4- Casting

54
Foot and Ankle Scored and Recorded Self-Assessment Examination 2015

Question 49 of 100

A 15-year-old high school basketball player has pain over a medial midfoot
prominence on his right foot. There has been no trauma and no specific treatment.
He has bilateral flexible pes planus and pain with inversion against resistance on
the right. His pain is disrupting or preventing his daily and sports activities.

1- Observation
2- Arizona brace
3- Medial arch support
4- Casting
5- Hindfoot fusion

PREFERRED RESPONSE: 4- Casting

Question 50 of 100

A 69-year-old woman has rigid painful left pes planus that has become less
symptomatic with casting. She has multiple comorbidities and is not a good
surgical candidate. She has failed a trial of activity without any supports.

1- Observation
2- Arizona brace
3- Medial arch support
4- Casting
5- Hindfoot fusion

PREFERRED RESPONSE: 2- Arizona brace

DISCUSSION

Treatment for pes planus revolves around 2 clinical parameters: pain and rigidity.
In the absence of pain, no intervention is warranted because there are no other
symptoms that can reasonably be linked to the foot shape. Flexible pes planus
(that corrects with heel rise) is usually normal and does not cause symptoms, but
it can be associated with a symptomatic accessory navicular, in which case the
patient may have pain over the medial navicular from either traction by the
tibialis posterior or the act of rubbing against the medial shoe counter. Rigid pes
planus is most frequently associated with a tarsal coalition, which classically
presents in late adolescence but can become symptomatic for the first time in

55
Foot and Ankle Scored and Recorded Self-Assessment Examination 2015

adults. The initial treatment for painful pes planus, whether flexible or rigid, is
immobilization, usually in a walking cast. This often is sufficient to relieve
symptoms on a permanent basis. Surgery should be contemplated only when this
treatment fails. Adult-acquired flatfoot is most commonly attributable to tibialis
posterior tendon dysfunction. In stage 3, the pes planus is rigid. If it is painful,
surgical treatment, which consists of a triple arthrodesis, may be considered.
However, if medical constraints or patient preference preclude surgery, an
Arizona brace can provide sufficient support to reduce symptoms to an acceptable
level to perform activities of daily living.

RECOMMENDED READINGS

Varner KE, Michelson JD. Tarsal coalition in adults. Foot Ankle Int. 2000
Aug;21(8):669-72. PubMed PMID: 10966365.
View Abstract at PubMed

Lin JL, Balbas J, Richardson EG. Results of non-surgical treatment of stage II


posterior tibial tendon dysfunction: a 7- to 10-year followup. Foot Ankle Int.
2008 Aug;29(8):781-6. doi: 10.3113/FAI.2008.0781. PubMed PMID: 18752775.
View Abstract at PubMed

Chao W, Wapner KL, Lee TH, Adams J, Hecht PJ. Nonoperative management of
posterior tibial tendon dysfunction. Foot Ankle Int. 1996 Dec;17(12):736-41.
PubMed PMID: 8973895.
View Abstract at PubMed

Cha SM, Shin HD, Kim KC, Lee JK. Simple excision vs the Kidner procedure for
type 2 accessory navicular associated with flatfoot in pediatric population. Foot
Ankle Int. 2013 Feb;34(2):167-72. doi: 10.1177/1071100712467616. Epub 2013
Jan 15. PubMed PMID: 23413054.
View Abstract at PubMed

Sullivan JA. Pediatric flatfoot: evaluation and management. J Am Acad Orthop


Surg. 1999 Jan;7(1):44-53. PubMed PMID: 9916191.
View Abstract at PubMed

56
Foot and Ankle Scored and Recorded Self-Assessment Examination 2015

Foot and Ankle Scored and Recorded Self-Assessment Examination 2015

Figure 51a Figure 51b Figure 51c

CLINICAL SITUATION FOR QUESTIONS 51 AND 52

Figure 51a demonstrates the sneaker wear pattern and Figures 51b and 51c are the
weight-bearing radiographs of a 20-year-old National Collegiate Athletic
Association Division I basketball player. Throughout his college career he has
experienced pain in the lateral aspect of his right foot. He has been treated with a
clamshell orthotic, but this preseason his pain is worse than ever. Upon
examination he has tenderness to palpation over the fifth metatarsal and his
peroneal strength is 5/5 bilaterally.

57
Foot and Ankle Scored and Recorded Self-Assessment Examination 2015

Question 51 of 100

What is the primary concern regarding resolution of this fracture?

1- Overload of the lateral foot column


2- Tendon pull on the fragments
3- Blood supply to this area
4- Patient activity level

PREFERRED RESPONSE: 3- Blood supply to this area

Question 52 of 100

Which treatment option will most reliably achieve long-term success?

1- Dwyer osteotomy
2- Surgical fixation with a solid screw
3- Surgical fixation with a cannulated screw
4- Iliac crest bone graft with plate fixation

PREFERRED RESPONSE: 2- Surgical fixation with a solid screw

DISCUSSION

There are 3 types of fifth metatarsal fractures. Zone 1 is an avulsion fracture.


Zone 2 fractures, also known as Jones fractures, occur in the watershed area of
the fifth metatarsal. A fracture must exit the intermetatarsal articulation between
the fourth and fifth metatarsals to be considered a Jones fracture. Zone 3 fractures
are distal to the articulation in the diaphysis of the fifth metatarsal. Jones fractures
are associated with a 15% to 20% nonunion rate with nonsurgical care. Surgical
intervention is preferred in athletic patients. Fixation with a solid screw is
mechanically stronger than fixation with a cannulated screw. Nonunions or failure
of hardware can be attributable to inadequate fixation or an unrecognized varus
heel alignment leading to lateral column overload.

RECOMMENDED READINGS

Zenios M, Kim WY, Sampath J, Muddu BN. Functional treatment of acute


metatarsal fractures: a prospective randomised comparison of management in a
cast versus elasticated support bandage. Injury. 2005 Jul;36(7):832-5. Epub 2005

58
Foot and Ankle Scored and Recorded Self-Assessment Examination 2015

Mar 21. PubMed PMID: 15949484.


View Abstract at PubMed

Fetzer GB, Wright RW. Metatarsal shaft fractures and fractures of the proximal
fifth metatarsal. Clin Sports Med. 2006 Jan;25(1):139-50, x. Review. PubMed
PMID: 16324980.
View Abstract at PubMed

Zwitser EW, Breederveld RS. Fractures of the fifth metatarsal; diagnosis and
treatment. Injury. 2010 Jun;41(6):555-62. doi: 10.1016/j.injury.2009.05.035.
Epub 2009 Jun 30. Review. PubMed PMID: 19570536.
View Abstract at PubMed

Hunt KJ, Goeb Y, Esparza R, Malone M, Shultz R, Matheson G. Site-Specific


Loading at the Fifth Metatarsal Base in Rehabilitative Devices: Implications for
Jones Fracture Treatment. PM R. 2014 May 28. pii: S1934-1482(14)00243-3. doi:
10.1016/j.pmrj.2014.05.011. [Epub ahead of print] PubMed PMID: 24880059.
View Abstract at PubMed

Nunley JA. Fractures of the base of the fifth metatarsal: the Jones fracture. Orthop
Clin North Am. 2001 Jan;32(1):171-80. Review. PubMed PMID: 11465126.
View Abstract at PubMed

Orr JD, Glisson RR, Nunley JA. Jones fracture fixation: a biomechanical
comparison of partially threaded screws versus tapered variable pitch screws. Am
J Sports Med. 2012 Mar;40(3):691-8. doi: 10.1177/0363546511428870. Epub
2012 Jan 6. PubMed PMID: 22227846.
View Abstract at PubMed

59
Foot and Ankle Scored and Recorded Self-Assessment Examination 2015

Foot and Ankle Scored and Recorded Self-Assessment Examination 2015

Figure 53

Question 53 of 100

Figure 53 is a coronal-cut CT scan of a 63-year-old woman who has a long-


standing pes planus. She is seen for lateral ankle discomfort. Upon examination
she is tender over the sinus tarsi and distal to the fibula. She has painless passive
hindfoot eversion with 5/5 eversion strength. The most appropriate diagnosis is

1- subtalar arthritis.
2- middle-facet coalition.
3- lateral impaction syndrome.
4- calcaneonavicular coalition.

PREFERRED RESPONSE: 3- lateral impaction syndrome.

DISCUSSION

With the use of CT scans, adults with symptomatic flatfoot deformity have been
noted to develop subluxation of the talocalcaneal joint with resulting lateral
hindfoot pain. Impingement of the talus and calcaneus in the sinus tarsi and/or
between the tip of the fibula and the calcaneus may occur. This impingement is
known as lateral impaction syndrome. Hindfoot motion is painless; therefore, this
patient does not have symptomatic subtalar arthritis. Middle facet and
calcaneonavicular coalitions are not present (hindfoot motion is present).

60
Foot and Ankle Scored and Recorded Self-Assessment Examination 2015

RECOMMENDED READINGS

Malicky ES, Crary JL, Houghton MJ, Agel J, Hansen ST Jr, Sangeorzan BJ.
Talocalcaneal and subfibular impingement in symptomatic flatfoot in adults. J
Bone Joint Surg Am. 2002 Nov;84-A(11):2005-9. PubMed PMID: 12429762. ?
View Abstract at PubMed

Ananthakrisnan D, Ching R, Tencer A, Hansen ST Jr, Sangeorzan BJ.


Subluxation of the talocalcaneal joint in adults who have symptomatic flatfoot. J
Bone Joint Surg Am. 1999 Aug;81(8):1147-54. PubMed PMID: 10466647. ?
View Abstract at PubMed

61
Foot and Ankle Scored and Recorded Self-Assessment Examination 2015

Foot and Ankle Scored and Recorded Self-Assessment Examination 2015

Figure 54

Question 54 of 100

Figure 54 is the lateral radiograph of a 55-year-old man who is evaluated for a 2-


year history of pain and stiffness of his right metatarsophalangeal (MTP) joint.
Upon examination he has dorsal bossing, severe crepitation, and pain with
passive range of motion. There is pain with the "grind" test. Dorsiflexion is
limited to 0 degrees. No sesamoid tenderness is present. What is the most
appropriate surgical treatment?

1- Chevron bunionectomy
2- Cheilectomy and removal of loose body
3- MTP arthrodesis
4- Resection of proximal phalanx

PREFERRED RESPONSE: 3- MTP arthrodesis

DISCUSSION

The radiograph reveals end-stage degenerative changes of the first MTP joint
with a dorsal loose body. MTP arthritis and decreased joint dorsiflexion is
referred to as hallux rigidus. A chevron bunionectomy is used to correct hallux
valgus deformity without arthritis. The cheilectomy is used in lesser degrees of
joint destruction. Resection of the proximal phalanx results in a floppy toe and is
generally not recommended.

62
Foot and Ankle Scored and Recorded Self-Assessment Examination 2015

RECOMMENDED READINGS

McNeil DS, Baumhauer JF, Glazebrook MA. Evidence-based analysis of the


efficacy for operative treatment of hallux rigidus. Foot Ankle Int. 2013
Jan;34(1):15-32. doi: 10.1177/1071100712460220. Review. PubMed PMID:
23386758.
View Abstract at PubMed

Deland JT, Williams BR. Surgical management of hallux rigidus. J Am Acad


Orthop Surg. 2012 Jun;20(6):347-58. doi: 10.5435/JAAOS-20-06-347. Review.
PubMed PMID: 22661564.
View Abstract at PubMed

63
Foot and Ankle Scored and Recorded Self-Assessment Examination 2015

Foot and Ankle Scored and Recorded Self-Assessment Examination 2015

Figure 55a Figure 55b

CLINICAL SITUATION FOR QUESTIONS 55 THROUGH 58

Figures 55a and 55b are the anteroposterior and lateral radiographs of a 57-year-
old man who fell off of a ladder 10 days ago and landed on his left foot. He is
now unable to weight bear on the left. He has no history of trauma to this foot,
and his medical history is unremarkable. Upon examination his left foot is
swollen and tender. Pulses and sensation are intact.

Question 55 of 100

What is the most likely diagnosis?

1- Fracture dislocation of the Lisfranc joint


2- Fracture dislocation of the Chopart joint
3- Fracture dislocation of the subtalar joint
4- Complex fracture dislocation of the midtarsal articulations

PREFERRED RESPONSE: 2- Fracture dislocation of the Chopart joint

64
Foot and Ankle Scored and Recorded Self-Assessment Examination 2015

Question 56 of 100

What is the most appropriate initial treatment?

1- Primary arthrodesis
2- Attempted closed reduction in the office
3- Attempted closed reduction under anesthesia with possible open reduction
4- Splinting to comfort level and progressive weight bearing as tolerated

PREFERRED RESPONSE: 3- Attempted closed reduction under anesthesia


with possible open reduction

Question 57 of 100

If a closed reduction is attempted, which structure most likely will prevent a


reduction?

1- Tibialis anterior tendon


2- Tibialis posterior tendon
3- Peroneus brevis tendon
4- Peroneus longus tendon

PREFERRED RESPONSE: 2- Tibialis posterior tendon

Question 58 of 100

After full healing from this injury, which option most likely will help to optimize
this patient's activities?

1- Extra-depth shoes
2- Rocker-bottom soles
3- Figure-of-8 soft-ankle brace
4- A patellar-tendon-bearing brace

PREFERRED RESPONSE: 2- Rocker-bottom soles

65
Foot and Ankle Scored and Recorded Self-Assessment Examination 2015

DISCUSSION

Midfoot fracture dislocations typically occur after a fall from a height or a motor
vehicle collision involving severe dorsiflexion loading of the foot from a brake
pedal or the floorboards. The deformity that results may be subtle because the
subluxation may be a valgus or varus rotation around the midfoot rather than pure
dorsiflexion. Early recognition and reduction is indicated to minimize secondary
complications such as nerve injury or vascular compromise. Closed reduction
usually necessitates formal anesthesia in an operating room to permit adequate
relaxation and reduction. In many cases, satisfactory reduction can be
accomplished closed, but the surgeon must be prepared to perform an open
reduction if needed. The most common reason for failed closed reduction is that
the talar head is caught by the tibialis posterior tendon (under which the talar head
has protruded). This acts as a Chinese finger trap, preventing relocation by the
usual distraction followed by a plantar flexion maneuver. In these cases, open
reduction is performed through a dorsomedial incision through which the tibialis
posterior is retracted medially, allowing reduction of the talonavicular joint. The
joints usually require pinning to maintain stability during healing. The long-term
prognosis for these injuries is guarded because many patients develop
degenerative changes in the Chopart joint. If symptomatic arthritis develops,
helpful external supports are designed to limit sagittal motion at the joint (for
example, carbon fiber inserts or rocker-bottom soles).

RECOMMENDED READINGS

Swords MP, Schramski M, Switzer K, Nemec S. Chopart fractures and


dislocations. Foot Ankle Clin. 2008 Dec;13(4):679-93, viii. Doi:
10.1016/j.fcl.2008.08.004. Review. PubMed PMID: 19013402.
View Abstract at PubMed

Richter M, Thermann H, Huefner T, Schmidt U, Goesling T, Krettek C. Chopart


joint fracture-dislocation: initial open reduction provides better outcome than
closed reduction. Foot Ankle Int. 2004 May;25(5):340-8. PubMed PMID:
15134617.
View Abstract at PubMed

66
Foot and Ankle Scored and Recorded Self-Assessment Examination 2015

Foot and Ankle Scored and Recorded Self-Assessment Examination 2015

Question 59 of 100

Which lower extremity muscle is first weakened in Charcot-Marie-Tooth (CMT)


disease?

1- Peroneus brevis
2- Peroneus longus
3- Foot intrinsics
4- Tibialis anterior

PREFERRED RESPONSE: 3- Foot intrinsics

DISCUSSION

Although many of the lower extremity muscles may be affected in CMT, those
innervated by the longest axons have been shown to be affected first. In the lower
extremity the muscles innervated by the longest axons are the intrinsic foot
muscles. The tibialis anterior and the peroneus brevis may be severely affected
but not before the foot intrinsics. The peroneus longus typically is spared,
resulting in the cavus.

RECOMMENDED READINGS

Pareyson D, Marchesi C. Diagnosis, natural history, and management of Charcot-


Marie-Tooth disease. Lancet Neurol. 2009 Jul;8(7):654-67. Review. PubMed
PMID: 19539237.
View Abstract at PubMed

Wenz W, Dreher T. Charcot-Marie-Tooth disease and the cavovarus foot. In:


Pinzur MS, ed. Orthopaedic Knowledge Update: Foot and Ankle 4. Rosemont,
IL: American Academy of Orthopaedic Surgeons; 2008:291-306.

67
Foot and Ankle Scored and Recorded Self-Assessment Examination 2015

Foot and Ankle Scored and Recorded Self-Assessment Examination 2015

Figure 60

CLINICAL SITUATION FOR QUESTIONS 60 THROUGH 63

Figure 60 is the standing radiograph of a 27-year-old man who played football


throughout his teen years. During those years, he noted that he had less mobility
of his left foot and ankle. He twisted his left foot and ankle 8 months ago and has
tried over-the-counter nonsteroidal anti-inflammatory drugs and a brace. He now
has pain and edema in the left sinus tarsi area. Upon examination the following
arcs of motion are identified: ankle dorsiflexion-right, 5 degrees/left, 5 degrees;
ankle plantar flexion-right, 30 degrees/left 30 degrees; foot inversion-right, 10
degrees/left, 5 degrees; foot eversion-right, 10 degrees/left 5 degrees.

Question 60 of 100

What are the most common locations of the pathology?

1- Calcaneocuboid and talocalcaneal joints


2- Talonavicular and calcaneocuboid joints
3- Talonavicular and calcaneonavicular joints
4- Talocalcaneal and calcaneonavicular joints

PREFERRED RESPONSE: 4- Talocalcaneal and calcaneonavicular joints

68
Foot and Ankle Scored and Recorded Self-Assessment Examination 2015

Question 61 of 100

What is the origin of this patient's pathology?

1- Traumatic
2- Genetic
3- Arthritic
4- Attritional

PREFERRED RESPONSE: 2- Genetic

Question 62 of 100

After a review of the images, it appears the appropriate next diagnostic step
should be

1- MRI.
2- CT scan.
3- bone scan.
4- ultrasound.

PREFERRED RESPONSE: 2- CT scan.

Question 63 of 100

The patient undergoes further testing and it is discovered that the lesion
encompasses 70% of the joint. What is the best next treatment option?

1- Arthroscopic debridement
2- Subtalar arthroereisis
3- Subtalar fusion
4- Lesion resection

PREFERRED RESPONSE: 3- Subtalar fusion

DISCUSSION

A tarsal coalition is an abnormal connection of 2 or more bones in the foot.


Although tarsal coalitions are present at birth, children and adults typically do not
69
Foot and Ankle Scored and Recorded Self-Assessment Examination 2015

show signs of the disorder until early adolescence or later. The exact incidence of
the disorder is hard to determine; however, it is caused by a gene mutation that
affects cells that produce the tarsal bones. The 2 most common locations for tarsal
coalitions are between the calcaneus and the navicular or between the talus and
the calcaneus. It is estimated that 1 out of every 100 people may have a tarsal
coalition. In 50% of cases, both feet are affected. Tarsal coalitions are rarely
discovered until symptoms arise. Symptoms may include stiff and painful feet, a
rigid flatfoot, or increased pain or a limp with high-level activities. Upon
examination, symptoms may include tenderness in the area of the coalition, loss
of motion, rigid flat feet, and arthritic changes of the joint. Imaging studies begin
with radiographs. A CT scan can provide bony detail for imaging tarsal coalitions
and determining the extent of the coalition and any accompanying degenerative
change. MRI can provide details of the soft tissues. Treatment includes
nonsurgical care including rest, orthotics, a temporary boot or cast, and injections.
Surgical options include resection with interposition of muscle or fatty tissue
from another area of the body or fusions when large (exceeding 50% of the joint),
more severe coalitions are encountered.

RECOMMENDED READINGS

Lemley F, Berlet G, Hill K, Philbin T, Isaac B, Lee T. Current concepts review:


Tarsal coalition. Foot Ankle Int. 2006 Dec;27(12):1163-9. Review. PubMed
PMID: 17207452.
View Abstract at PubMed

Badgley CE. Coalition of the calcaneus and the navicular. Arch Surg.
1927;15:75-88.

Conway JJ, Cowell HR. Tarsal coalition: clinical significance and


roentgenographic demonstration. Radiology. 1969 Mar;92(4):799-811. PubMed
PMID: 5767760.
View Abstract at PubMed

Herzenberg JE, Goldner JL, Martinez S, Silverman PM. Computerized


tomography of talocalcaneal tarsal coalition: a clinical and anatomic study. Foot
Ankle. 1986 Jun;6(6):273-88. PubMed PMID: 3721364.
View Abstract at PubMed

Wechsler RJ, Schweitzer ME, Deely DM, Horn BD, Pizzutillo PD. Tarsal
coalition: depiction and characterization with CT and MR imaging. Radiology.
1994 Nov;193(2):447-52. PubMed PMID: 7972761.
View Abstract at PubMed

70
Foot and Ankle Scored and Recorded Self-Assessment Examination 2015

Leonard MA. The inheritance of tarsal coalition and its relationship to spastic flat
foot. J Bone Joint Surg Br. 1974 Aug;56B(3):520-6. PubMed PMID: 4421359.
View Abstract at PubMed

71
Foot and Ankle Scored and Recorded Self-Assessment Examination 2015

Foot and Ankle Scored and Recorded Self-Assessment Examination 2015

Figure 64c
Figure 64b
Figure 64a

Question 64 of 100

Figures 64a through 64c are the MR images and radiograph of an active 30-year-
old man who has been treated for pain in his subtalar joint for 6 months. He has
had casting, physical therapy, and bracing but continues to have activity-limiting
pain. An injection into the subtalar joint under fluoroscopic guidance temporarily
relieved his pain. His best surgical option at this time is

1- resection of the coalition bar.


2- subtalar fusion.
3- medial sliding calcaneal osteotomy.
4- Dwyer osteotomy.

PREFERRED RESPONSE: 2- subtalar fusion.

DISCUSSION

When contemplating the causes of subtalar joint degeneration in young patients,


an unstable tarsal coalition should be considered in the absence of antecedent
trauma. Initial treatment with casting is appropriate because this intervention can
relieve symptoms for many patients. There are 2 surgical options for a
symptomatic tarsal coalition: bar resection or completion fusion. Risk factors for
a poor outcome after bar resection are adult age and a bar that encompasses more
than 50% of the middle facet of the subtalar joint. Because this patient has both
risk factors, the appropriate procedure is a subtalar fusion.

72
Foot and Ankle Scored and Recorded Self-Assessment Examination 2015

RECOMMENDED READINGS

Comfort TK, Johnson LO. Resection for symptomatic talocalcaneal coalition. J


Pediatr Orthop. 1998 May-Jun;18(3):283-8. PubMed PMID: 9600549.
View Abstract at PubMed

Thorpe SW, Wukich DK. Tarsal coalitions in the adult population: does treatment
differ from the adolescent? Foot Ankle Clin. 2012 Jun;17(2):195-204. doi:
10.1016/j.fcl.2012.03.004. Epub 2012 Apr 6. Review. PubMed PMID: 22541520.
View Abstract at PubMed

73
Foot and Ankle Scored and Recorded Self-Assessment Examination 2015

Foot and Ankle Scored and Recorded Self-Assessment Examination 2015

Question 65 of 100

What is a risk factor for interdigital neuroma?

1- Female gender
2- Increased mobility between the third and fourth rays
3- Achilles tendon contracture
4- Prolonged standing at work

PREFERRED RESPONSE: 1- Female gender

DISCUSSION

The only proven risk factor for development of an interdigital neuroma is female
gender, which likely is related to the use of fashionable shoes that force plantar
flexion of the metatarsal heads and secondary hyperdorsiflexion of the
metatarsophalangeal joints. The other factors listed have not been proven to cause
interdigital neuroma, as well as mediolateral compression of the forefoot.

RECOMMENDED READINGS

Hill KJ. Peripheral nerve disorder. In: Pinzur MS, ed. Orthopaedic Knowledge
Update: Foot and Ankle 4. Rosemont, IL: American Academy of Orthopaedic
Surgeons; 2008:307-327.

Schon LC, Mann RA. Diseases of the nerves. In: Coughlin MJ, Mann RA,
Saltzman CL, eds. Surgery of the Foot and Ankle. 8th ed. Philadelphia, PA:
Mosby-Elsev

74
Foot and Ankle Scored and Recorded Self-Assessment Examination 2015

Foot and Ankle Scored and Recorded Self-Assessment Examination 2015

Question 66 of 100

Which Morton neuroma histology is most common?

1- Perineural fibrosis
2- Wallerian degeneration
3- Distal axonopathy
4- Segmental demyelination

PREFERRED RESPONSE: 1- Perineural fibrosis

DISCUSSION

Morton neuroma is a compressive neuropathy of the interdigital nerves of the


forefoot that most commonly is noted in the third web space. Perineural fibrosis is
commonly noted on microscopic examination of resected tissue. This accounts
for the relatively higher failure rate of neuroma decompression by transection of
the intermetatarsal ligament as opposed to resection. Wallerian degeneration is
seen following axonal transection in the distal part of the nerve. Distal
axonopathy is degeneration of the axon and myelin and is associated with
"stocking-glove" distribution neuropathy. Segmental demyelination is
characterized by breakdown of myelin with an intact axon and is associated with
lead poisoning and hereditary sensory motor neuropathy.

RECOMMENDED READINGS

Akermark C, Crone H, Saartok T, Zuber Z. Plantar versus dorsal incision in the


treatment of primary intermetatarsal Morton's neuroma. Foot Ankle Int. 2008
Feb;29(2):136-41. doi: 10.3113/FAI.2008.0136. PubMed PMID: 18315967.
View Abstract at PubMed

Ha'Eri GB, Fornasier VL, Schatzker J. Morton's neuroma--pathogenesis and


ultrastructure. Clin Orthop Relat Res. 1979 Jun;(141):256-9. PubMed PMID:
477115.
View Abstract at PubMed

Graham CE, Graham DM. Morton's neuroma: a microscopic evaluation. Foot


Ankle. 1984 Nov-Dec;5(3):150-3. PubMed PMID: 6519606.
View Abstract at PubMed

75
Foot and Ankle Scored and Recorded Self-Assessment Examination 2015

Foot and Ankle Scored and Recorded Self-Assessment Examination 2015

Question 67 of 100

A patient sustained a puncture wound to the plantar aspect of his foot. He was
wearing shoes and socks at the time of the injury. Systemic antibiotic
administration with specific coverage for which bacterial species (in addition to
Staphylococcus aureus) should be instituted?

1- Escherichia coli
2- Mycobacterium marinum
3- Pseudomonas
4- Clostridium

PREFERRED RESPONSE: 3- Pseudomonas

DISCUSSION

Puncture wounds sustained through a shoe and sock increase risk for
Pseudomonas infection. Clostridium are associated with soil-contaminated
wounds. Mycobacterium marinum is associated with injuries sustained within
water.

RECOMMENDED READINGS

DeCoster TA, Miller RA. Management of Traumatic Foot Wounds. J Am Acad


Orthop Surg. 1994 Jul;2(4):226-230. PubMed PMID: 10709013.
View Abstract at PubMed

Raikin SM. Common infections of the foot. In: Richardson EG, ed. Orthopaedic
Knowledge Update: Foot and Ankle 3. Rosemont, IL: American Academy of
Orthopaedic Surgeons; 2004:199-205.

76
Foot and Ankle Scored and Recorded Self-Assessment Examination 2015

Foot and Ankle Scored and Recorded Self-Assessment Examination 2015

Figure 68

Question 68 of 100

Figure 68 is the radiograph of a 33-year-old runner who recently decided to begin


running barefoot on trails. Since his transition to running without shoes 3 months
ago, he has been having pain in the second metatarsophalangeal (MTP) joint. He
feels like he is walking on a stone, notes edema in the ball of his foot, and has
started to see a deviation of the second toe. What is the most likely etiology of
these symptoms and findings?

1- Tear of the plantar plate


2- Second MTP joint synovitis
3- Second metatarsal stress fracture
4- Flexor tendonitis

PREFERRED RESPONSE: 1- Tear of the plantar plate

DISCUSSION

Lesser-toe plantar plate injuries are becoming increasingly recognized. Patients


typically have an increase in pain, a positive Lachman test result upon
examination, and deviation of the MTP joint. On radiograph, MTP subluxation
can be appreciated. Nonsurgical treatment with a metatarsal pad may be

77
Foot and Ankle Scored and Recorded Self-Assessment Examination 2015

attempted. Many patients who have surgery will have a partial or full tear of the
plantar plate. The repair necessitates reinsertion of the plantar plate to the base of
the proximal phalanx.

RECOMMENDED READINGS

Doty JF, Coughlin MJ, Weil L Jr, Nery C. Etiology and management of lesser toe
metatarsophalangeal joint instability. Foot Ankle Clin. 2014 Sep;19(3):385-405.
doi: 10.1016/j.fcl.2014.06.013. Epub 2014 Jul 10. PubMed PMID: 25129351.
View Abstract at PubMed

Nery C, Coughlin MJ, Baumfeld D, Raduan FC, Mann TS, Catena F. Prospective
evaluation of protocol for surgical treatment of lesser MTP joint plantar plate
tears. Foot Ankle Int. 2014 Sep;35(9):876-85. doi: 10.1177/1071100714539659.
Epub 2014 Jun 23. PubMed PMID: 24958766.
View Abstract at PubMed

Chalayon O, Chertman C, Guss AD, Saltzman CL, Nickisch F, Bachus KN. Role
of plantar plate and surgical reconstruction techniques on static stability of lesser
metatarsophalangeal joints: a biomechanical study. Foot Ankle Int. 2013
Oct;34(10):1436-42. doi: 10.1177/1071100713491728. Epub 2013 Jun 17.
PubMed PMID: 23774466.
View Abstract at PubMed

78
Foot and Ankle Scored and Recorded Self-Assessment Examination 2015

Foot and Ankle Scored and Recorded Self-Assessment Examination 2015

Figure 69

Question 69 of 100

Figure 69 is the radiograph of a 9-year-old who has posterior hindfoot pain while
running. What is the most likely diagnosis?

1- Kohler disease
2- Thiemann disease
3- Freiberg infraction
4- Sever disease

PREFERRED RESPONSE: 4- Sever disease

DISCUSSION

Sever disease is a traction apophysitis at the Achilles tendon insertion on the


calcaneus. Plain radiographs may be unremarkable for this condition. Other
osteochondroses of the foot include Kohler disease (tarsal navicular), Freiberg
infraction (lesser metatarsal head), and Thiemann disease (great toe phalanx).

RECOMMENDED READINGS

Sullivan RJ. Adolescent foot and ankle conditions. In: Pinzur MD, ED.
Orthopaedic Knowledge Update: Foot and Ankle 4. Rosemont, IL: American
Academy of Orthopaedic Surgeons; 2008:47-55.

Feldman DS. Osteochondrosis. In: Spivak JM, Di Cesare PE, Feldman Ds, et al,
eds. Orthopaedic: A Study Guide. New York, NY: McGraw-Hill; 1999:765-766.

79
Foot and Ankle Scored and Recorded Self-Assessment Examination 2015

Schantz K, Rasmussen F. Thiemann's finger or toe disease. Follow-up of seven


cases. Acta Orthop Scand. 1986 Feb;57(1):91-3. PubMed PMID: 3962642.
View Abstract at PubMed

80
Foot and Ankle Scored and Recorded Self-Assessment Examination 2015

Examination Center
Foot and Ankle Scored and Recorded Self-Assessment Examination 2015

Question 70 of 100

A 62-year-old runner injured his right ankle 8 weeks ago. He has ongoing lateral
ankle pain and swelling that did not improve with 4 weeks of immobilization and
4 weeks of physical therapy. MR images demonstrate a longitudinal tear of the
peroneus brevis tendon. Treatment should involve

1- peroneus brevis tendon repair.


2- peroneus longus to peroneus brevis tenodesis.
3- 6 weeks in short-leg cast.
4- platelet-rich plasma injection.

PREFERRED RESPONSE: 1- peroneus brevis tendon repair.

DISCUSSION

Additional nonsurgical treatment is not beneficial for peroneal tendon tears. Early
repair of longitudinal tears reduces risk for progression to a full-thickness tear
that would necessitate peroneus brevis to peroneus longus tenodesis. Platelet-rich
plasma has no role in peroneal tendon tears.

RECOMMENDED READINGS

Arbab D, Tingart M, Frank D, Abbara-Czardybon M, Waizy H, Wingenfeld C.


Treatment of isolated peroneus longus tears and a review of the literature. Foot
Ankle Spec. 2014 Apr;7(2):113-8. doi: 10.1177/1938640013514273. Epub 2013
Dec 30. Review. PubMed PMID: 24381076.
View Abstract at PubMed

Coughlin MJ, Schon LC. Disorders of tendons. In: Coughlin MJ, Saltzman CL,
Anderson RB, eds. Mann's Surgery of the Foot and Ankle. 9th ed. Philadelphia,
PA: Elsevier-Saunders; 2014: 1188-1291.

81
Foot and Ankle Scored and Recorded Self-Assessment Examination 2015

Foot and Ankle Scored and Recorded Self-Assessment Examination 2015

Figure 71 Figure 72 Figure 74

RESPONSES FOR QUESTIONS 71 THROUGH 75

1- Lapidus procedure
2- First metatarsophalangeal (MTP) joint arthrodesis and lesser metatarsal head
resection
3- Proximal first metatarsal osteotomy
4- Distal first metatarsal chevron osteotomy
5- Biplanar distal first metatarsal osteotomy

Match the appropriate surgical procedure listed above with the clinical scenario
described below.

82
Foot and Ankle Scored and Recorded Self-Assessment Examination 2015

Question 71 of 100

Figure 71 is the radiograph of a 67-year-old patient with rheumatoid arthritis who


has experienced 6 months of increasing pain, swelling, and foot deformity. Anti-
inflammatory medications, orthotics, and extra-depth shoes fail to provide
sufficient relief.

1- Lapidus procedure
2- First metatarsophalangeal (MTP) joint arthrodesis and lesser metatarsal head
resection
3- Proximal first metatarsal osteotomy
4- Distal first metatarsal chevron osteotomy
5- Biplanar distal first metatarsal osteotomy

PREFERRED RESPONSE: 2- First metatarsophalangeal (MTP) joint


arthrodesis and lesser metatarsal head resection

Question 72 of 100

Figure 72 is the radiograph of a 58-year-old active woman with progressive


medial first MTP joint pain, particularly with shoe wear and increased activity.
The patient has no hypermobility.

1- Lapidus procedure
2- First metatarsophalangeal (MTP) joint arthrodesis and lesser metatarsal head
resection
3- Proximal first metatarsal osteotomy
4- Distal first metatarsal chevron osteotomy
5- Biplanar distal first metatarsal osteotomy

PREFERRED RESPONSE: 3- Proximal first metatarsal osteotomy

83
Foot and Ankle Scored and Recorded Self-Assessment Examination 2015

Question 73 of 100

A 44-year-old patient who has had a proximal first metatarsal osteotomy has
recurrent pain and difficulty wearing many types of shoes. Radiographs show a
large 1-2 intermetatarsal angle (IMA).

1- Lapidus procedure
2- First metatarsophalangeal (MTP) joint arthrodesis and lesser metatarsal head
resection
3- Proximal first metatarsal osteotomy
4- Distal first metatarsal chevron osteotomy
5- Biplanar distal first metatarsal osteotomy

PREFERRED RESPONSE: 1- Lapidus procedure

Question 74 of 100

Figure 74 is the radiograph of a 31-year-old woman with a painful bunion


deformity that has failed nonsurgical treatment.

1- Lapidus procedure
2- First metatarsophalangeal (MTP) joint arthrodesis and lesser metatarsal head
resection
3- Proximal first metatarsal osteotomy
4- Distal first metatarsal chevron osteotomy
5- Biplanar distal first metatarsal osteotomy

PREFERRED RESPONSE: 4- Distal first metatarsal chevron osteotomy

84
Foot and Ankle Scored and Recorded Self-Assessment Examination 2015

Question 75 of 100

A 25-year-old man has had a painful bunion deformity since childhood.


Nonsurgical treatment has failed. Weight-bearing radiographs reveal a congruent
hallux valgus (HV) with a large distal metatarsal articular angle (DMAA) and a
normal 1-2 IMA. There is no arthritis or first tarsometatarsal joint or
hypermobility.

1- Lapidus procedure
2- First metatarsophalangeal (MTP) joint arthrodesis and lesser metatarsal head
resection
3- Proximal first metatarsal osteotomy
4- Distal first metatarsal chevron osteotomy
5- Biplanar distal first metatarsal osteotomy

PREFERRED RESPONSE: 5- Biplanar distal first metatarsal osteotomy

DISCUSSION

Most cases of HV should initially be treated nonsurgically. When indicated, the


decision to pursue surgical treatment of HV is dependent on a variety of factors
including presence of arthritis, joint stability/ligamentous laxity, congruence of
the first MTP joint, and severity of deformity. Mild congruent HV may be treated
with a distal chevron osteotomy. A distal soft-tissue procedure may be added if
the deformity is incongruent. Moderate to severe HV deformities may be treated
with proximal first metatarsal osteotomies such as the crescentic, Ludloff, or scarf
osteotomy. Larger HV deformities are almost always associated with incongruent
HV angles, and they necessitate a distal soft-tissue procedure to realign the first
MTP joint. Fusion of the first MTP joint is indicated in cases of severe HV
deformity or associated osteoarthritis or rheumatoid arthritis. The Lapidus
procedure, an arthrodesis of the first metatarsal-medial cuneiform joint, has been
a success when used to treat patients with first-ray instability (hypermobility) or
as a salvage procedure for those who have bunion recurrence after previous
surgery. A biplanar distal chevron osteotomy is indicated for congruent HV
deformities involving a large DMAA (the angle measured between the slope of
the articular surface of the first metatarsal head and the longitudinal axis of the
first metatarsal).

RECOMMENDED READINGS

Coughlin MJ, Jones CP. Hallux valgus and first ray mobility. A prospective
study. J Bone Joint Surg Am. 2007 Sep;89(9):1887-98. PubMed PMID:
85
Foot and Ankle Scored and Recorded Self-Assessment Examination 2015

17768183.
View Abstract at PubMed

Robinson AH, Limbers JP. Modern concepts in the treatment of hallux valgus. J
Bone Joint Surg Br. 2005 Aug;87(8):1038-45. Review. PubMed PMID:
16049235.
View Abstract at PubMed

Coetzee JC, Wickum D. The Lapidus procedure: a prospective cohort outcome


study. Foot Ankle Int. 2004 Aug;25(8):526-31. PubMed PMID: 15363372.
View Abstract at PubMed

Coughlin MJ, Mann RA. Hallux valgus. In: Coughlin MJ, Mann RA, Saltzman
CL, eds. Surgery of the Foot and Ankle. 8th ed. Philadelphia, PA: Mosby-
Elsevier; 2007:2

86
Foot and Ankle Scored and Recorded Self-Assessment Examination 2015

Foot and Ankle Scored and Recorded Self-Assessment Examination 2015

Figure 76a Figure 76b Figure 76c

CLINICAL SITUATION FOR QUESTIONS 76 THROUGH 79

Figures 76a through 76c are the clinical photograph and radiographs of a 48-year-
old man with a 10-year history of diabetes who has an open wound on his left
foot. He had neither previous trauma nor earlier similar occurrences. His diabetes
is well controlled, with a hemoglobin A1c level of 6.7% (reference range, 4%-
7%). His examination is notable for well-perfused feet with intact pulses and
intact sensation to light touch. There is mild tenderness over the left forefoot. The
erythema in the forefoot does not resolve with elevation. The head of the fifth
metatarsal can be probed through the wound.

Question 76 of 100

What is the most common underlying etiology for this condition in this clinical
setting?

1- Microvascular disease
2- Poorly controlled blood glucose
3- Sensory neuropathy
4- Chronic renal failure

PREFERRED RESPONSE: 3- Sensory neuropathy

87
Foot and Ankle Scored and Recorded Self-Assessment Examination 2015

Question 77 of 100

If nonsurgical treatment of this wound fails, the most likely cause for failure is

1- poor tolerance to casting.


2- poorly controlled blood glucose.
3- inappropriate choice of antibiotics.
4- underlying osteomyelitis.

PREFERRED RESPONSE: 4- underlying osteomyelitis.

Question 78 of 100

If surgery is necessary, what is the most appropriate procedure?

1- Below-knee amputation
2- Debridement of all infected tissue with primary closure
3- Debridement of all infected tissue with free-flap closure
4- Debridement of all infected tissue and a negative-pressure dressing for the
resulting wound

PREFERRED RESPONSE: 4- Debridement of all infected tissue and a


negative-pressure dressing for the resulting wound

Question 79 of 100

After the patient's wound has healed, which intervention is critical to prevent
future ulceration?

1- A well-fitted prosthesis on the left


2- Making sure he wears large shoes to decrease pressure on his feet
3- A bilateral ankle-foot orthoses fitting
4- Custom-molded soft shoe inserts

PREFERRED RESPONSE: 4- Custom-molded soft shoe inserts

88
Foot and Ankle Scored and Recorded Self-Assessment Examination 2015

DISCUSSION

The most common etiology for plantar ulcers in patients with diabetes is
neuropathy; 80% of these patients have adequate vascularity. The pathobiology is
increased pressure applied on skin that lacks protective sensation (detected by
lack of sensation to Semmes-Weinstein monofilament 5.07). The presence of
sensation to light touch is not predictive of ulcer risk, but absent Semmes-
Weinstein sensation at 5.07 carries a 30% risk for developing an ulcer. Intact light
touch sensation does not rule out loss of protective sensation, and, therefore, does
not rule out sensory neuropathy. The grading of an ulcer primarily depends on the
existence of deep infection, either an abscess or osteomyelitis. In the presence of
deep infection, formal surgical debridement is required. The surgery should be
limited to the tissues that are compromised, without an attempt at primary closure
(which increases risk for recurrent infection). Negative-pressure dressings are a
significant advance in postsurgical treatment by permitting delayed healing
without further surgical intervention. Once healed, the key to preventing recurrent
ulceration is the use of orthopaedic shoes with custom-molded soft inserts that
accommodate the contours of the feet. This is particularly true when surgery has
removed a portion of the foot; the foot otherwise will shift in a shoe that does not
include a custom molded insert incorporating a filler that occupies the space left
by the surgical resection.

RECOMMENDED READINGS

Pham H, Armstrong DG, Harvey C, Harkless LB, Giurini JM, Veves A.


Screening techniques to identify people at high risk for diabetic foot ulceration: a
prospective multicenter trial. Diabetes Care. 2000 May;23(5):606-11. PubMed
PMID: 10834417.
View Abstract at PubMed

Pinzur MS, Slovenkai MP, Trepman E, Shields NN; Diabetes Committee of


American Orthopaedic Foot and Ankle Society. Guidelines for diabetic foot care:
recommendations endorsed by the Diabetes Committee of the American
Orthopaedic Foot and Ankle Society. Foot Ankle Int. 2005 Jan;26(1):113-9.
PubMed PMID: 15680122.
View Abstract at PubMed

Chantelau E. Effectiveness of diabetic therapeutic footwear in preventing


reulceration: response to Maciejewski et al. Diabetes Care. 2004
Dec;27(12):3024-5; author reply 3025-6. PubMed PMID: 15562237.
View Abstract at PubMed

89
Foot and Ankle Scored and Recorded Self-Assessment Examination 2015

Foot and Ankle Scored and Recorded Self-Assessment Examination 2015

Question 80 of 100

Which repair technique for an osteochondral lesion of the medial talus shoulder
produces hyaline cartilage that is similar to native cartilage and will not degrade
over time?

1- Autologous osteochondral transplantation


2- Arthroscopic bone marrow stimulation
3- Chondroplasty
4- Osteochondral allograft transplantation

PREFERRED RESPONSE: 1- Autologous osteochondral transplantation

DISCUSSION

Autologous osteochondral transplantation (typically involving tubular grafts


harvested from the knee) has been shown to replace a talar defect with viable
hyaline cartilage. The results over the medium term show good clinical outcomes,
and MRI studies reveal cartilage repair similar to native cartilage. Chondroplasty
and arthroscopic bone-marrow stimulation are both associated with good clinical
results for smaller lesions, but these techniques develop fibrocartilaginous repair
tissue composed of type I collagen instead of hyaline cartilage. Osteochondral
transplantation of fresh allografts performed less than 14 days after harvest
contains high chondrocyte viability. Few clinical studies report long-term results,
but radiographic studies demonstrate high rates of collapse and resorption. Joint
space narrowing has been noted in 60% of ankles treated with bulk grafts after an
average of 44 months. Autologous chondrocyte implantation (both periosteum-
covered and matrix-associated techniques) has been shown to create hyaline
cartilage in some studies, but fibrocartilage creation has been reported in others.

RECOMMENDED READINGS

Schachter AK, Chen AL, Reddy PD, Tejwani NC. Osteochondral lesions of the
talus. J Am Acad Orthop Surg. 2005 May-Jun;13(3):152-8. Review. PubMed
PMID: 15938604. .
View Abstract at PubMed

Mitchell ME, Giza E, Sullivan MR. Cartilage transplantation techniques for talar
cartilage lesions. J Am Acad Orthop Surg. 2009 Jul;17(7):407-14. Review.

90
Foot and Ankle Scored and Recorded Self-Assessment Examination 2015

PubMed PMID: 19571296.


View Abstract at PubMed

Murawski CD, Kennedy JG. Operative treatment of osteochondral lesions of the


talus. J Bone Joint Surg Am. 2013 Jun 5;95(11):1045-54. doi:
10.2106/JBJS.L.00773. Review. PubMed PMID: 23780543.
View Abstract at PubMed

91
Foot and Ankle Scored and Recorded Self-Assessment Examination 2015

Foot and Ankle Scored and Recorded Self-Assessment Examination 2015

Question 81 of 100

What is the most common turf toe mechanism of injury?

1- Hyperdorsiflexion of the first metatarsophalangeal (MTP) joint and axial load


with the foot fixed in equinus
2- Hyperdorsiflexion of the first MTP joint with valgus thrust
3- Hyperplantarflexion of the first MTP joint axial load with the foot fixed in
equinus
4- Hyperplantarflexion of the first MTP joint with valgus thrust

PREFERRED RESPONSE: 1- Hyperdorsiflexion of the first


metatarsophalangeal (MTP) joint and axial load with the foot fixed in equinus

DISCUSSION

Turf toe, or capsuloligamentous injury to the first MTP joint, most commonly is
caused by an axial load being applied to a fixed, dorsiflexed great toe with the
heel off the ground. The external force causes further dorsiflexion of the great toe,
leading to injury to the capsuloligamentous complex. A less common mechanism
of turf toe is hyperplantarflexion of the great toe with valgus stress, which is seen
in beach volleyball players.

RECOMMENDED READINGS

Kadakia AR, Molloy A. Current concepts review: traumatic disorders of the first
metatarsophalangeal joint and sesamoid complex. Foot Ankle Int. 2011
Aug;32(8):834-9. Review. PubMed PMID: 22049873.
View Abstract at PubMed

Clanton TO, McGarvey W. Athletic Injuries to the soft tissues of the foot and
ankle. In: Coughlin MJ, Mann RA, Saltzman CL, eds. Surgery of the Foot and
Ankle. 8th ed. Philadelphia, PA: Mosby-Elsevier; 2007:1526-1535

92
Foot and Ankle Scored and Recorded Self-Assessment Examination 2015

Foot and Ankle Scored and Recorded Self-Assessment Examination 2015

Figure 82a Figure 82b

Question 82 of 100

Figures 82a and 82b are the clinical photograph and radiograph of a 60-year-old
man with a 30-year history of diabetes complicated by borderline chronic renal
failure, heart failure controlled by medication, and bilateral lower extremity
neuropathy. He is currently wheelchair bound because of his cardiopulmonary
limitations, but uses his legs for transfers. He has had a progressive left ankle
deformity that has progressed to the point at which he cannot use his leg for pivot
transfers. He is adamant that something should be done to improve his living
situation. Which surgical option can best achieve his goal of using the leg for
transfers?

1- Local debridement of the ankle with long-term bracing


2- Attempted fusion using a multiplanar external fixation
3- Attempted fusion using an intramedullary rod
4- Attempted fusion using screw fixation

PREFERRED RESPONSE: 3- Attempted fusion using an intramedullary rod

DISCUSSION

The indications for surgery in a Charcot joint are an unbraceable deformity or


recurrent ulceration despite use of adequate bracing. Any significant varus/valgus
deformity of the ankle or hindfoot (as in this patient) is unbraceable, whereas
93
Foot and Ankle Scored and Recorded Self-Assessment Examination 2015

midfoot Charcot deformities usually can be treated nonsurgically. The goal of


surgery is to provide a stable aligned foot and ankle to permit activities and
reduce risk for recurrent ulceration. This does not mean that there has to be a
radiographic fusion because many patients with diabetes achieve a stable
nonunion without pain. The only surgical construct that will provide long-term
inherent stability and alignment in this situation is use of a hindfoot fusion rod. It
provides better stability and purchase than screw fixation and can be left in
indefinitely (unlike an external fixator).

RECOMMENDED READINGS

Dalla Paola L, Volpe A, Varotto D, Postorino A, Brocco E, Senesi A, Merico M,


De Vido D, Da Ros R, Assaloni R. Use of a retrograde nail for ankle arthrodesis
in Charcot neuroarthropathy: a limb salvage procedure. Foot Ankle Int. 2007
Sep;28(9):967-70. PubMed PMID: 17880869.
View Abstract at PubMed

Pinzur MS. Benchmark analysis of diabetic patients with neuropathic (Charcot)


foot deformity. Foot Ankle Int. 1999 Sep;20(9):564-7. PubMed PMID:
10509683.
View Abstract at PubMed

Pinzur M. Surgical versus accommodative treatment for Charcot arthropathy of


the midfoot. Foot Ankle Int. 2004 Aug;25(8):545-9. PubMed PMID: 15363375.
View Abstract at PubMed

94
Foot and Ankle Scored and Recorded Self-Assessment Examination 2015

Foot and Ankle Scored and Recorded Self-Assessment Examination 2015

Figure 83a Figure 83b

CLINICAL SITUATION FOR QUESTIONS 83 THROUGH 87

Figures 83a and 83b are the clinical photographs of a 42-year-old woman with a
BMI of 31 who has had a 1-year history of right heel pain.

Question 83 of 100

Pain and tenderness at location 2 on Figures 83a and 83b is most consistent with
which diagnosis?

1- Baxter's nerve entrapment.


2- Plantar fasciitis.
3- Tarsal tunnel syndrome.
4- Fat pad atrophy.

PREFERRED RESPONSE: 2- Plantar fasciitis.

95
Foot and Ankle Scored and Recorded Self-Assessment Examination 2015

Question 84 of 100

What is the most appropriate initial diagnostic test for plantar heel pain?

1- MRI
2- Ultrasound
3- Weight-bearing plain radiographs
4- Inflammatory arthritis serology

PREFERRED RESPONSE: 3- Weight-bearing plain radiographs

Question 85 of 100

What is the strongest risk factor for plantar fasciitis?

1- Female gender
2- African American ancestry
3- BMI higher than 30
4- Weight lifting and body building

PREFERRED RESPONSE: 3- BMI higher than 30

Question 86 of 100

What is the most reliable nonsurgical treatment for plantar fasciitis?

1- High-impact loading exercise


2- Corticosteroid injection
3- Platelet-rich plasma (PRP) injection
4- Plantar fascia stretching

PREFERRED RESPONSE: 4- Plantar fascia stretching

96
Foot and Ankle Scored and Recorded Self-Assessment Examination 2015

Question 87 of 100

Which surgical procedure should be considered for treatment of chronic plantar


fasciitis?

1- Endoscopic or open plantar fasciotomy


2- Heel spur excision
3- Achilles tendon lengthening
4- Extensile approach, medial and plantar, to include release of tarsal tunnel and
complete plantar fasciotomy

PREFERRED RESPONSE: 1- Endoscopic or open plantar fasciotomy

DISCUSSION

Imaging studies in the evaluation of plantar fasciitis should always include


weight-bearing foot radiographs to reveal alignment and exclude calcaneal stress
fracture, tumor, subtalar arthritis, and insertional posterior spurs. MRI is
occasionally indicated in problematic cases. Ultrasound can be helpful to evaluate
thickening and disease in the proximal plantar fascia. Ultrasound is quick and
much more cost effective than MRI. Laboratory screenings to evaluate
inflammatory arthritis are indicated only for patients with bilateral heel pain who
may be more likely to have systemic disease.

In the nonsurgical treatment of plantar fasciitis, high-impact loading exercises


may make the condition worse. Corticosteroid injections may provide short-term
relief only and can occasionally cause plantar fascia rupture. They should be used
with caution. PRP injections are expensive and currently not covered by
insurance. Studies have not demonstrated long-term pain relief with PRP. Plantar
fascia-specific stretching has been shown more effective than Achilles tendon
stretching alone.

Surgical treatment is indicated for fewer than 5% of patients. It is not necessary to


resect the heel spur because the spur is not attached to the plantar fascia and
rarely contributes to a patient's pain. The open extensile approach is associated
with a much longer recovery than the open or endoscopic approaches and is no
longer justified. Multiple studies have demonstrated the efficacy of endoscopic
and open plantar fasciotomy techniques.

97
Foot and Ankle Scored and Recorded Self-Assessment Examination 2015

RECOMMENDED READINGS

Bader L, Park K, Gu Y, O'Malley MJ. Functional outcome of endoscopic plantar


fasciotomy. Foot Ankle Int. 2012 Jan;33(1):37-43. doi: 10.3113/FAI.2012.0037.
PubMed PMID: 22381234.
View Abstract at PubMed

Lareau CR, Sawyer GA, Wang JH, DiGiovanni CW. Plantar and medial heel
pain: diagnosis and management. J Am Acad Orthop Surg. 2014 Jun;22(6):372-
80. doi: 10.5435/JAAOS-22-06-372. PubMed PMID: 24860133.
View Abstract at PubMed

98
Foot and Ankle Scored and Recorded Self-Assessment Examination 2015

Foot and Ankle Scored and Recorded Self-Assessment Examination 2015

Question 88 of 100

A 45-year-old woman with grade II adult-acquired flatfoot deformity has pain on


the lateral side of her foot just distal to the tip of the fibula. Which component of
a comprehensive flatfoot reconstruction most likely will address the deformity
responsible for this pain?

1- Spring ligament reconstruction


2- Lateral column lengthening
3- Medial-displacing calcaneal osteotomy
4- Medial cuneiform opening-wedge osteotomy

PREFERRED RESPONSE: 3- Medial-displacing calcaneal osteotomy

DISCUSSION

Patients develop lateral ankle pain with progression of adult-acquired flatfoot


deformity. This is associated with increased hindfoot valgus deformity. Calcaneal
fibular impingement has been considered the primary cause of this pain. Studies
demonstrate that arthrosis of the posterior facet of the subtalar joint strongly
correlates with lateral pain in adult-acquired flatfoot deformity. Both conditions
are related to hindfoot valgus deformity. Although lateral column lengthening is a
powerful tool for correction of flatfoot deformity, its effect on hindfoot deformity
is less defined. Lateral column lengthening provides better correction of the
longitudinal arch of the midfoot and realignment of the medial column than other
osteotomies. A medializing calcaneal osteotomy has a significant linear effect on
hindfoot valgus alignment. Spring ligament reconstruction and medial cuneiform
opening-wedge osteotomies have less effect on hindfoot alignment than the
medial calcaneal slide.

RECOMMENDED READINGS

Ellis SJ, Deyer T, Williams BR, Yu JC, Lehto S, Maderazo A, Pavlov H, Deland
JT. Assessment of lateral hindfoot pain in acquired flatfoot deformity using
weightbearing multiplanar imaging. Foot Ankle Int. 2010 May;31(5):361-71. doi:
10.3113/FAI.2010.0361. PubMed PMID: 20460061.
View Abstract at PubMed

Chan JY, Williams BR, Nair P, Young E, Sofka C, Deland JT, Ellis SJ. The
contribution of medializing calcaneal osteotomy on hindfoot alignment in the
99
Foot and Ankle Scored and Recorded Self-Assessment Examination 2015

reconstruction of the stage II adult acquired flatfoot deformity. Foot Ankle Int.
2013 Feb;34(2):159-66. doi: 10.1177/1071100712460225. Epub 2013 Jan 10.
PubMed PMID: 23413053.
View Abstract at PubMed

100
Foot and Ankle Scored and Recorded Self-Assessment Examination 2015

Foot and Ankle Scored and Recorded Self-Assessment Examination 2015

Figure 89a Figure 89b Figure 89c

Question 89 of 100

Figures 89a through 89c are the radiographs of a 35-year-old woman who has had
7 years of progressive ankle pain. She experiences stiffness and pain despite the
use of an ankle-foot orthosis. Examination reveals pain along the anterior
tibiotalar joint without tenderness to the subtalar or talonavicular joints. What is
the most appropriate surgical intervention?

1- Total ankle arthroplasty (TAA)


2- Ankle arthrodesis
3- Tibiotalocalcaneal arthrodesis
4- Anterior tibial exostectomy with Achilles lengthening

PREFERRED RESPONSE: 2- Ankle arthrodesis

DISCUSSION

This patient has isolated posttraumatic ankle arthritis with significantly decreased
ankle range of motion that is best treated with an isolated ankle arthrodesis to
eliminate pain. Because this patient is younger than 50 years of age and has
limited presurgical range of motion, she is not a candidate for ankle arthroplasty.
Additionally, TAA outcomes among patients who have a posttraumatic etiology

101
Foot and Ankle Scored and Recorded Self-Assessment Examination 2015

are worse than for those with osteoarthritis. Tibiotalocalcaneal arthrodesis is


suited for patients with associated subtalar arthritis or rigid hindfoot deformity
that necessitates correction. Anterior tibial exostectomy will not resolve the
underlying arthritic pain and is not indicated.

RECOMMENDED READINGS

Easley ME, Adams SB Jr, Hembree WC, DeOrio JK. Results of total ankle
arthroplasty. J Bone Joint Surg Am. 2011 Aug 3;93(15):1455-68. doi:
10.2106/JBJS.J.00126. Review. PubMed PMID: 21915552.
View Abstract at PubMed

Spirt AA, Assal M, Hansen ST Jr. Complications and failure after total ankle
arthroplasty. J Bone Joint Surg Am. 2004 Jun;86-A(6):1172-8. PubMed PMID:
15173289.
View Abstract at PubMed

Ajis A, Henriquez H, Myerson M. Postoperative range of motion trends following


total ankle arthroplasty. Foot Ankle Int. 2013 May;34(5):645-56. doi:
10.1177/1071100713481433. Epub 2013 Mar 11. PubMed PMID: 23478890.
View Abstract at PubMed

Thomas R, Daniels TR, Parker K. Gait analysis and functional outcomes


following ankle arthrodesis for isolated ankle arthritis. J Bone Joint Surg Am.
2006 Mar;88(3):526-35. PubMed PMID: 16510818.
View Abstract at PubMed

Coester LM, Saltzman CL, Leupold J, Pontarelli W. Long-term results following


ankle arthrodesis for post-traumatic arthritis. J Bone Joint Surg Am. 2001 Feb;83-
A(2):219-28. PubMed PMID: 11216683.
View Abstract at PubMed

102
Foot and Ankle Scored and Recorded Self-Assessment Examination 2015

Foot and Ankle Scored and Recorded Self-Assessment Examination 2015

Question 90 of 100

Which structure is the primary stabilizer of the lesser metatarsophalangeal (MTP)


joint?

1- Flexor digitorum brevis


2- Flexor digitorum longus
3- Plantar plate
4- Collateral ligament

PREFERRED RESPONSE: 3- Plantar plate

DISCUSSION

Synovitis of the second MTP occurs in association with instability of the joint.
This can be idiopathic or secondary to an external deforming force (such as a
hallux valgus or shoe wear causing a claw toe). The primary stabilizer of the MTP
joint for translation in the vertical plane is the plantar plate. The flexor digitorum
brevis flexes the MTP joint, and the flexor digitorum longus flexes all joints of
the toe. The collateral ligaments are primary stabilizers in the transverse plane
and secondary stabilizers in the sagittal plane.

RECOMMENDED READINGS

Deland JT, Sung IH. The medial crosssover toe: a cadaveric dissection. Foot
Ankle Int. 2000 May;21(5):375-8. PubMed PMID: 10830654.
View Abstract at PubMed

Suero EM, Meyers KN, Bohne WH. Stability of the metatarsophalangeal joint of
the lesser toes: a cadaveric study. J Orthop Res. 2012 Dec;30(12):1995-8. doi:
10.1002/jor.22173. Epub 2012 Jun 13. PubMed PMID: 22696467.
View Abstract at PubMed

103
Foot and Ankle Scored and Recorded Self-Assessment Examination 2015

Foot and Ankle Scored and Recorded Self-Assessment Examination 2015

Figure 91a Figure 91b Figure 91c

CLINICAL SITUATION FOR QUESTIONS 91 THROUGH 95

Figure 91a is the sagittal plane ultrasound of the second web space of a 48-year-
old woman who has noted burning pain in the ball of her right foot for 2 years.
The pain intermittently radiates into her second and the third toes. Figure 91b
shows the surgical procedure and Figure 91c is the pathologic specimen.

Question 91 of 100

The cause of this patient's symptoms most likely is

1- pigmented villonodular synovitis (PVNS).


2- lipoma.
3- ganglion.
4- interdigital neuroma.

PREFERRED RESPONSE: 4- interdigital neuroma.

Question 92 of 100

The most appropriate pedorthic management of symptomatic interdigital neuroma


involves

1- a metatarsal pad.
2- a dancer's pad.
3- a custom foot orthosis.
4- a medial heel wedge.

PREFERRED RESPONSE: 1- a metatarsal pad.

104
Foot and Ankle Scored and Recorded Self-Assessment Examination 2015

Question 93 of 100

Open or endoscopic decompression of interdigital neuroma is predicated on the


hypothesis that interdigital neuromas represent a compression neuropathy from
which anatomic structure?

1- Lumbrical tendon
2- Interosseous muscle
3- Intermetatarsal ligament
4- Plantar plate

PREFERRED RESPONSE: 3- Intermetatarsal ligament

Question 94 of 100

The primary advantage of the plantar approach for resection of interdigital


neuromas is

1- a less painful scar.


2- a lower infection rate.
3- better access to the neuroma.
4- earlier weight bearing.

PREFERRED RESPONSE: 3- better access to the neuroma.

Question 95 of 100

The most common complication associated with corticosteroid injection for the
treatment of interdigital neuroma is

1- Infection
2- Chronic regional pain syndrome (CRPS)
3- Hammertoe deformity
4- Hives

PREFERRED RESPONSE: 3- Hammertoe deformity

105
Foot and Ankle Scored and Recorded Self-Assessment Examination 2015

DISCUSSION

Interdigital neuromas represent perineural fibrosis of the intermetatarsal nerve


most commonly seen in the second and third web spaces. The photographs depict
a neuroma in the second web space, which is less common than in the third space.
Its hypoechoic appearance is characteristic on ultrasound. PVNS occurs in joints.
Lipomas are fatty tumors. A ganglion cyst can occasionally occur adjacent to a
nerve, but it has a cystic appearance.

The most popular theory on the etiology of interdigital neuromas is compression


neuropathy by the intermetatarsal ligament. The plantar approach for the resection
of interdigital neuromas has become more popular, especially for recurrent
neuromas. The major advantage is better visualization and access to the
neuromas.

Infection and allergic reactions such as hives are rare after steroid injection.
CRPS is not known to occur in this setting. Hammertoe formation can occur,
especially after multiple steroid injections into an interdigital neuroma.

RECOMMENDED READINGS

Schon LC, Reed MA. Disorder of the nerves. In: Coughlin MJ, Saltzman CL,
Anderson RB, eds. Mann's Surgery of the Foot and Ankle. 9th ed. Philadelphia,
PA: Elsevier-Saunders; 2014:622-641.

Gauthier G. Thomas Morton's disease: a nerve entrapment syndrome. A new


surgical technique. Clin Orthop Relat Res. 1979 Jul-Aug;(142):90-2. PubMed
PMID: 498653.
View Abstract at PubMed

106
Foot and Ankle Scored and Recorded Self-Assessment Examination 2015

Foot and Ankle Scored and Recorded Self-Assessment Examination 2015

Figure 96

CLINICAL SITUATION FOR QUESTIONS 96 THROUGH 98

Figure 96 is the anteroposterior projection foot radiograph of a 35-year-old


woman who is injured in a high-speed motorcycle collision. Her assessment
reveals an isolated injury. Her pulses are palpable and her neuromotor
examination is intact. There is tenting of skin with ecchymosis over the second
metatarsal. The foot is extremely swollen but not tense. Passive toe range of
motion is painless.

Question 96 of 100

The most appropriate urgent orthopaedic intervention involves

1- splinting in situ with serial compartment checks.


2- urgent closed reduction and splinting with serial compartment checks.
3- closed reduction and percutaneous fixation when the operating room is
available.
4- emergency foot fasciotomy.

PREFERRED RESPONSE: 2- urgent closed reduction and splinting with serial


compartment checks.

107
Foot and Ankle Scored and Recorded Self-Assessment Examination 2015

Question 97 of 100

Six months after surgical repair, the patient reports that when tapping over her
central midfoot incision, paresthesias to the third and fourth toes occur. What is
the most likely diagnosis?

1- Complex regional pain syndrome (CRPS)


2- Laceration of the deep peroneal nerve
3- Neuritis of the superficial peroneal nerve
4- Scar hypersensitivity

PREFERRED RESPONSE: 3- Neuritis of the superficial peroneal nerve

Question 98 of 100

The patient returns 1 year later to report curling of her toes and numbness on the
plantar surface of her foot. What is the most likely cause of this condition?

1- CRPS
2- Plantar fasciitis
3- Plantar fibromatosis
4- Subclinical compartment syndrome

PREFERRED RESPONSE: 4- Subclinical compartment syndrome

DISCUSSION

Radiographs reveal a Lisfranc fracture dislocation with fractures of the first and
second metatarsals. Tenting of skin that is over a bony prominence is an
orthopaedic emergency. The fracture dislocation should be reduced without delay.
There is no evidence of compartment syndrome of the foot, but this may develop
and monitoring is necessary. Toe deformity may develop on a delayed basis
because of the subclinical presentation. Nerve irritation is not uncommon with
dorsal midfoot surgical incisions. A positive Tinel test result over the midfoot in
the distribution of the superficial common peroneal nerve is consistent with a
stretch injury to this nerve. CRPS is usually associated with multiple nerve
distributions and autonomic nerve findings such as cold hypersensitivity and
hyperhidrosis.

108
Foot and Ankle Scored and Recorded Self-Assessment Examination 2015

RECOMMENDED READINGS

Benirschke SK, Meinberg EG, Anderson SA, Jones CB, Cole PA. Fractures and
dislocations of the midfoot: Lisfranc and Chopart injuries. Instr Course Lect.
2013;62:79-91. PubMed PMID: 23395016.
View Abstract at PubMed

Schepers T, Oprel PP, Van Lieshout EM. Influence of approach and implant on
reduction accuracy and stability in lisfranc fracture-dislocation at the
tarsometatarsal joint. Foot Ankle Int. 2013 May;34(5):705-10. doi:
10.1177/1071100712468581. Epub 2013 Jan 14. PubMed PMID: 23637239.
View Abstract at PubMed

109
Foot and Ankle Scored and Recorded Self-Assessment Examination 2015

Foot and Ankle Scored and Recorded Self-Assessment Examination 2015

Figure 99

Question 99 of 100

Figure 99 is the radiograph of an 18-year-old National Collegiate Athletic


Association Division I basketball player who jumped for a basket. After landing,
he was unable to put weight on his left great toe. He developed pain, swelling,
and ecchymosis maximally around the sesamoids. When assessing stability of the
first metatarsophalangeal joint, he appears to have more laxity on the left. What is
the best next step?

1- Standing radiographs of the right foot


2- MRI of the foot
3- Carbon fiber insole with a Morton extension
4- Surgery for reconstruction

PREFERRED RESPONSE: 1- Standing radiographs of the right foot

DISCUSSION

This patient jumped on his forefoot and landed with pain in his great toe. His
radiograph shows possible increased space between the sesamoids and the base of
the phalanx. A comparison radiograph on the other side will reveal if this position
is normal for this patient. If findings are asymmetric, turf toe injury is a
possibility based on this patient's symptoms and mechanism of injury.

110
Foot and Ankle Scored and Recorded Self-Assessment Examination 2015

RECOMMENDED READINGS

Waldrop NE 3rd, Zirker CA, Wijdicks CA, Laprade RF, Clanton TO.
Radiographic evaluation of plantar plate injury: an in vitro biomechanical study.
Foot Ankle Int. 2013 Mar;34(3):403-8. doi: 10.1177/1071100712464953. Epub
2013 Jan 14. PubMed PMID: 23520299.
View Abstract at PubMed

McCormick JJ, Anderson RB. The great toe: failed turf toe, chronic turf toe, and
complicated sesamoid injuries. Foot Ankle Clin. 2009 Jun;14(2):135-50. doi:
10.1016/j.fcl.2009.01.001. Review. PubMed PMID: 19501799.
View Abstract at PubMed

111
Foot and Ankle Scored and Recorded Self-Assessment Examination 2015

Foot and Ankle Scored and Recorded Self-Assessment Examination 2015

Video 100

Question 100 of 100

Video 100 is the presurgical lateral ankle examination of a 45-year-old woman


who has had pain and discomfort for 2 years along the posterolateral ankle
following a sudden dorsiflexion injury. She notes occasional clicking and
popping, and she has not experienced resolution of her symptoms despite
immobilization and physical therapy. Examination reveals a stable ankle-to-
anterior drawer and inversion stress testing. No strength deficit is noted, but she
has apprehension with resisted eversion. MR images do not reveal evidence of
tendonosis or tear. The most appropriate surgical intervention is

1- imbrication of the lateral collateral ligaments with reinforcement with the


extensor retinaculum (modified Brostrom procedure).
2- peroneal tendon synovectomy and tubularization of the peroneus brevis.
3- groove deepening of the fibula with imbrication of the peroneal retinaculum.
4- excision of the peroneus brevis with tenodesis of the proximal stump to the
peroneus longus.

PREFERRED RESPONSE: 3- groove deepening of the fibula with imbrication


of the peroneal retinaculum.

DISCUSSION

This patient has a clear history of dorsiflexion injury complicated by chronic


peroneal tendon dislocation. The symptoms and findings are consistent with
dislocation in this particular case. Groove deepening of the posterior fibula with
associated imbrication of the peroneal retinaculum is the most effective surgical
procedure. Associated synovitis or tendonosis should be addressed. However,
failure to deepen the groove and imbricate the retinaculum will result in
continued discomfort. Consequently, both responses that involve isolated tendon
surgery are not appropriate. Associated subjective instability can be noted in these
patients. The examination is critical to determine the stability of the lateral
collateral complex, which is intact in this case (so imbrication is not indicated). A
sense of apprehension is a common examination finding because patients sense
that the peroneals will subluxate with resisted eversion. Placement of the
examiner's hands on the peroneals to stabilize the tendons should relieve this

112
Foot and Ankle Scored and Recorded Self-Assessment Examination 2015

apprehension. A patient may not be able to voluntarily dislocate the tendon.


Dynamic ultrasound is the most sensitive radiographic examination for detection
of dislocation. Intrasheath peroneal subluxation may also occur and is treated
similarly.

RECOMMENDED READINGS

Raikin SM. Intrasheath subluxation of the peroneal tendons. Surgical technique. J


Bone Joint Surg Am. 2009 Mar 1;91 Suppl 2 Pt 1:146-55. doi:
10.2106/JBJS.H.01356. PubMed PMID: 19255207.
View Abstract at PubMed

Philbin TM, Landis GS, Smith B. Peroneal tendon injuries. J Am Acad Orthop
Surg. 2009 May;17(5):306-17. Review. PubMed PMID: 19411642.
View Abstract at PubMed

Ogawa BK, Thordarson DB. Current concepts review: peroneal tendon


subluxation and dislocation. Foot Ankle Int. 2007 Sep;28(9):1034-40. Review.
PubMed PMID: 17880883.
View Abstract at PubMed

This is the last question of the exam.

113

You might also like